Section 3 Cardiology

SECTION 3 Cardiology 48 CHAPTER 1 49 Bradycardia 50 Case: An 87-year-old man found down An 87-year-old man with a history of coronary artery disease, hypertension, and hyperlipidemia is admitted to the hospital after bathroom where she discovered her husband unresponsive on the hospital for further evaluation. He does not recall any details of the being found down at home. The patient’s wife heard a thud in the floor. The patient regained consciousness and was brought into the event. He feels light-headed but otherwise has no complaints. Heart rate is 42 beats per minute, and blood pressure is 85/47 mm Hg. On examination, the pulse is regular and slow. Electrocardiogram (ECG) is shown in Figure 1-1. FIGURE 1-1 What rhythm disturbance is present in this patient? What is the path of electrical conduction in the normal heart? In the normal heart, an impulse spontaneously originates from the sinoatrial (SA) node, which is located in the subepicardial surface at the junction of the right atrium and superior vena cava. The impulse propagates through the myocytes of the right and left atria simultaneously before reaching the Atrioventricular (AV) node, which is located in the inferior portion of the right atrium. From there, the impulse is conducted to the bundle of His within the membranous septum, which then separates into the right and left bundle branches supplying the right and left ventricles, respectively (Figure 1-2) 51 FIGURE 1-2 The cardiac conduction system. A, Cardiac conduction system anatomy. B, Action potentials of specific cardiac cells. C, Relationship of surface electrocardiogram to the action potential. Which main coronary artery supplies the SA node in most patients? Which main coronary artery supplies the AV node in most patients? How is heart rate regulated? What is the definition of bradycardia in adults? What is the average drop in heart rate during sleep in young healthy patients and in the elderly? What is the relationship between cardiac output and heart rate? What are the symptoms of bradycardia? What are the physical findings of bradycardia? What are the 2 main electrocardiographic categories of bradycardia? What is the definition of a wide QRS complex? The small boxes on the electrocardiogram represent how many milliseconds? The SA nodal artery originates from the proximal right coronary artery in 65% of patients and the circumflex in 25%; it arises from both in 10% of patients.1 The AV nodal artery originates from the right coronary artery in 80% of patients and the circumflex in 10%; it arises from both in 10% of patients.1 The sympathetic and parasympathetic nervous systems innervate the conduction system of the heart. Parasympathetic tone decreases SA node automaticity and AV node conduction, whereas sympathetic input increases SA node automaticity and AV node conduction.1 The average resting heart rate in adults is 70 beats per minute. Bradycardia is classically defined as a heart rate <60 beats per minute, but there is considerable variation in normal between populations. Age, fitness, and other clinical factors must be taken into consideration.1,2 During sleep, heart rate in young healthy patients decreases by an average of 24 beats per minute. In the elderly, heart rate decreases by an average of 14 beats per minute.3,4 Cardiac output (CO) is equal to the forward stroke volume (SV) of the left ventricle per beat multiplied by heart rate (HR).1CO =SV ×HR Patients with bradycardia may be asymptomatic. Symptoms may include fatigue, weakness, light-headedness, and syncope.1 The cardinal physical finding of bradycardia is a slow pulse rate, which can be regular or irregular. Additional findings may include hypotension, cool extremities, and cannon A waves (in the setting of AV dissociation). Bradycardia can be associated with a narrow QRS complex or a wide QRS complex. A wide QRS complex is defined electrocardiographically as QRS duration >120 ms (see Figure 1-2). At the standard paper speed of 25 mm/s, each small box (1 mm in width) on the ECG corresponds to 40 ms. Each large box, which is composed of 5 small boxes, represents 200 ms (see Figure 1-2). 52 Narrow-Complex Bradycardia What are the 2 electrocardiographic subcategories of narrow-complex bradycardia? What are the electrocardiographic characteristics of a regular rhythm? Narrow-complex bradycardia can be associated with a regular rhythm or an irregular rhythm. Regular rhythm is defined electrocardiographically by the presence of QRS complexes that are separated by a constant interval (ie, the R-R interval is constant). 53 Narrow-Complex Bradycardia WITH REGULAR RHYTHM What are the causes of narrow-complex bradycardia with regular rhythm? A 19-year-old woman with anorexia nervosa and a heart rate of 48 beats per minute. Electrocardiographic sawtooth pattern. Sinus bradycardia. Atrial flutter with AV block and slow ventricular rate (Figure 1-3). FIGURE 1-3 An example of atrial flutter with 3:1 AV conduction, resulting in bradycardia. Note This underlying atrial rhythm results in tachycardia when conduction through the AV node is 1:1 and is characterized by the presence of P waves before every QRS complex that are often inverted in leads II, III, and aVF. No visible or discernible P waves before 
the QRS complexes. What are the causes of sinus bradycardia? What medications are associated with sinus bradycardia? What distinctive pulse-temperature pattern can be observed in some patients with infectious causes of sinus bradycardia? What is the mechanism of sinus bradycardia in healthy athletes? In the setting of atrial flutter with AV block, assuming an atrial rate of 300 beats per minute, what ratio of AV conduction will produce a bradycardic rhythm? What is the definition of atrial tachycardia? What is the typical rate of a junctional escape rhythm? sawtooth-shaped flutter waves (arrows) with an atrial rate <300 beats per minute. (From De Fer TM. The Washington Manual of Outpatient Internal Medicine. Philadelphia, PA: Wolters Kluwer; 2015.) Atrial tachycardia with AV block and slow ventricular rate. Junctional escape rhythm. Causes of sinus bradycardia include conditions intrinsic to the SA node, such as idiopathic degeneration (aging), ischemia, infiltrative disorders (eg, amyloidosis), collagen vascular diseases (eg, systemic lupus erythematosus), infectious diseases (eg, Chagas disease), myotonic dystrophy, and surgical trauma (eg, valve replacement), as well as conditions extrinsic to the SA node, including medications (eg, β-blockers), electrolyte disturbances (eg, hypokalemia), neurally- mediated reflexes (eg, carotid sinus hypersensitivity), hypothyroidism, and hypothermia.1 Medications commonly associated with sinus bradycardia include β-blockers, calcium channel blockers, digoxin, clonidine, and antiarrhythmic agents (eg, amiodarone).1 Some infections are associated with pulse-temperature dissociation, known as “relative bradycardia.” Normally, for each degree Farenheit increase in temperature, there is a corresponding increase in heart rate of 10 beats per minute. Infections associated with relative bradycardia include legionella, psittacosis, Q fever, typhoid fever, typhus, babesiosis, malaria, leptospirosis, yellow fever, dengue fever, viral hemorrhagic fevers, and Rocky Mountain spotted fever.5 Well-trained athletes develop sinus bradycardia as a result of Increased vagal tone.1 Assuming an atrial rate of 300 beats per minute, bradycardia is produced when atrial flutter is associated with AV block of at least 5:1 (which corresponds to a ventricular rate of 60 beats per minute). When the atrial rate is <300 beats per minute, ratios of AV block <5:1 may produce bradycardia (see Figure 1-3). Atrial tachycardia is an atrial rhythm with an atrial rate >100 beats per minute that does not originate in the sinus node (sinus rhythm is associated with P waves that are identical in morphology and upright in leads I and aVF).6 Junctional escape rhythms are associated with a heart rate of 40 to 60 beats per minute; these rhythms are variably responsive to alterations in autonomic tone and pharmacologic agents. Junctional escape rhythms may originate more distally in the conduction system (eg, in the fascicles or distal Purkinje fibers); these escape rhythms have a wide QRS morphology and slower heart rates.1 54 55 Narrow-Complex Bradycardia with Irregular Rhythm What are the causes of narrow-complex bradycardia with Irregular Rhythm? Bradycardia during expiration only. Sinus node intermittently stops firing. Sinus node fires normally, but the impulses are intermittently blocked from depolarizing the atria. No P waves. A rhythm that most commonly originates within the right atrium. The following electrocardiographic features are observed in a patient with narrow-complex bradycardia with Irregular Rhythm: 
P waves occur regularly at a rate of 120 beats per minute; every QRS complex is preceded by a P wave, but not all P waves are followed by a QRS complex; and P waves are inverted in leads II, III, and aVF. Intermittently dropped QRS complexes. What is sinus arrhythmia? What is sinus arrest? What is sinoatrial exit block? Sinus arrhythmia. Sinus arrest (a type of sinus pause due to impulse generation failure in the SA node). Sinoatrial exit block (a type of sinus pause due to impulse transmission failure). Atrial fibrillation with slow ventricular rate Atrial flutter with variable AV block and slow ventricular rate. Atrial tachycardia with variable AV block and slow ventricular rate. Mobitz type I (Wenckebach) and Mobitz type II second-degree AV blocks. Sinus arrhythmia describes the normal reflex-mediated variability in heart rate during the respiratory cycle in which the heart rate is faster during inspiration and slower during expiration. It may provide certain physiologic advantages, such as enhanced gas exchange by improving ventilation-perfusion matching.7 Sinus arrest is the intermittent failure of the sinus node to generate an impulse (ie, generator failure). The duration of the pause has no relationship to the basic underlying sinus rate, which helps differentiate this entity from second-degree sinoatrial exit block (in which the duration of the pause is a multiple of the basic underlying sinus rate). Sinus arrest longer than 3 seconds in duration requires careful assessment to identify symptomatic correlations and may require intervention (eg, pacemaker placement).8 Sinoatrial exit block is the delay or failure of sinus node impulses to propagate through the SA node to neighboring atrial tissue (ie, transmission failure). There are several subtypes. First-degree sinoatrial exit block describes delayed conduction out of the sinus node. It cannot be readily identified electrocardiographically. Second-degree sinoatrial exit block—of which there are 2 subtypes—describes periodic failure of sinus node impulses to exit the sinus node. In second-degree sinoatrial exit block type I, the dropped P wave is preceded by progressively shorter P-P intervals. In second-degree sinoatrial exit block type II, the duration of the pause is a whole number multiple of the immediately preceding P-P interval, a distinctive electrocardiographic finding (Figure 1-4). Third-degree sinoatrial exit block describes complete failure of sinus node impulses to exit the nodal region. Regardless of the subtype, treatment with placement of a permanent pacemaker is usually indicated for symptomatic patients.8 56 FIGURE 1-4 Second-degree sinoatrial exit block type II (lead II). P waves are regular except after the What are the causes of slow ventricular rate in the setting of atrial fibrillation? In the setting of atrial flutter with AV block, assuming an atrial rate of 300 beats per minute, what ventricular rates would be expected with the following ratios of AV conduction: 2:1, 3:1, 4:1, and 5:1? What is the prognosis of atrial tachycardia? Which type of AV block is characterized electrocardiographically by progressive PR interval lengthening followed by a nonconducted P wave? How often is Mobitz II second-degree AV block associated with a wide QRS complex on ECG? third QRS complex, where a P wave is dropped. The first P wave after the pause occurs when it would have been expected had there been no interruption in the regular sinus P waves (dashed arrows), a characteristic feature of second-degree sinoatrial exit block type II. (From Katz AM. Physiology of the Heart. 5th ed. Philadelphia, PA: Lippincott Williams & Wilkins; 2011.) Atrial fibrillation with slow ventricular rate can occur as a result of medication (eg, β-blocker), conduction-system disease (eg, AV block), and Increased vagal tone (eg, trained athletes).9 Atrial flutter with an atrial rate of 300 beats per minute and 2:1, 3:1, 4:1, and 5:1 AV conduction will produce ventricular rates of 150, 100, 75, and 60 beats per minute, respectively. Atrial tachycardia occurs most commonly in patients without heart disease and usually follows a benign course.10 Mobitz I second-degree AV block is characterized by nonconducted (or dropped) beats preceded by progressively lengthening AV conduction intervals (most easily determined on ECG by measuring the PR intervals before and after the nonconducted beat) (see Figure 3-3). Mobitz II second-degree AV block is associated with a wide QRS complex in 80% of cases; the complex is narrow in the remaining 20% (see Figure 3-4).11 57 Wide-Complex Bradycardia What are the 2 electrocardiographic subcategories of wide- complex bradycardia? Wide-complex bradycardia can be associated with a regular rhythm or an irregular rhythm. 58 Wide-Complex Bradycardia WITH REGULAR RHYTHM What are the causes of wide-complex bradycardia with regular rhythm? A baseline wide QRS complex. A 34-year-old man hospitalized with aortic valve endocarditis develops light-headedness and severe bradycardia and is found to have cannon A waves on assessment of the jugular venous waveform. What electrocardiographic feature can distinguish sinus bradycardia with bundle branch block from a ventricular escape rhythm? What is the typical rate of a ventricular escape rhythm? Regular supraventricular rhythm (eg, sinus rhythm) with a baseline wide QRS complex (ie, bundle branch block) and slow ventricular rate. Complete heart block with ventricular escape rhythm. The presence of AV association (ie, all QRS complexes are preceded by a P wave; all P waves are followed by QRS complexes in regular intervals) would be expected in a patient with sinus bradycardia with a baseline wide QRS complex. Ventricular escape rhythms are characterized by AV dissociation. Ventricular escape rhythms from complete heart block typically occur at a rate between 20 and 40 beats per minute (see Figure 3-6).12-14 59 Wide-Complex Bradycardia with Irregular Rhythm What are the causes of wide-complex bradycardia with Irregular Rhythm? A baseline wide QRS complex. A 66-year-old woman with symptomatic AV block that worsens with exercise. What is the most likely underlying rhythm if the ECG shows irregular bradycardia with flutter waves and a wide QRS complex? Why is Mobitz II second-degree AV block associated with a wide QRS complex? Can bradycardia occur in pacemaker- dependent patients? What are some clinical scenarios that might result in relative bradycardia in a pacemaker-dependent patient? Irregular supraventricular rhythm (eg, atrial fibrillation) with a baseline wide QRS complex (ie, bundle branch block) and slow ventricular rate. Mobitz II second-degree AV block. Atrial flutter with variable AV block, a baseline wide QRS complex, and slow ventricular rate would present as irregular bradycardia with flutter waves and a wide QRS complex. Patients with Mobitz II second-degree AV block usually have preexisting bundle branch blocks (causing a baseline wide QRS complex); the nonconducted beats occur with intermittent failure of the remaining bundle branch.11,12 Pacemakers are programmed to set a “lower rate limit,” below which the heart rate should not drop. A drop in heart rate below the lower rate limit can be seen in pacemaker or pacer lead malfunction. Additionally, heart rates that do not meet the definition of bradycardia (ie, rates above 60 beats per minute) in a pacemaker-dependent patient can be considered relative bradycardia in clinical scenarios in which the rate is slower than that expected for hemodynamic needs (eg, sepsis). To keep up with the metabolic demand, the heart rate on the pacemaker may need to be Increased in patients with sepsis, anemia, hypoxemia, and low cardiac output. 60 Case Summary An 87-year-old man with a history of coronary artery disease, hypertension, and hyperlipidemia presents with syncope and is found to have symptomatic bradycardia. What rhythm disturbance is present in this patient? Junctional escape rhythm. 61 Bonus Questions What electrocardiographic The ECG in this case (see Figure 1-1) demonstrates narrow-complex bradycardia with regular rhythm, which narrows the differential diagnosis to sinus features in this case are bradycardia, atrial flutter with AV block and slow ventricular rate, atrial tachycardia with AV block and slow ventricular rate, and junctional escape suggestive of a junctional rhythm.iThe absence of clearly discernible P waves with 1:1 conduction or flutter waves suggests a junctional escape rhythm. If no P waves arerlying rhythm? discernible, the underlying rhythm is likely sinus arrest with junctional escape. If P waves are discernible but without 1:1 conduction, the unde rhythm s most likely sinus with complete heart block and junctional escape, or sinus slowing with isorhythmic AV dissociation. What other major ic finding is The ECG in this case (see Figure 1-1) demonstrates ST-segment elevation involving the inferior leads, consistent with acute inferior myocardial present in this case? electrocardiograph infarction. What is the most likely causem The junctional escape rhythm in this case is most likely the result of an inferior myocardial infarctioniin the distribution of the right coronary artery, of the junctional escape rhyth which supplies the SA and AV nodes. Possible underlying conduction disturbances include sinus arrest, sinus bradycardia, and heart block. Because in this case? appropriately timed P waves are not discernible on the ECG (see Figure 1-1), the underlying conduct on disturbance cannot be determined. What immediate treatment red Treatment with intravenous fluids, medications, transcutaneous or transvenous temporary pacing, and coronary artery reperfusion strategies (eg, What long-term treatment o If symptomatic bradycardia persists in the patient in this case, placement of a permanent pacemaker should be considered.1 in this case? strategies should be conside percutaneous coronary intervention or fibrinolytic agents) should be considered in this case. strategy should be offered t the patient in this case if the underlying cause is not reversible and symptomatic bradycardia persists? 62 Key Points <60 beats per minute, but there is considerable variation in normal Cardiac output is equal to the forward stroke volume of the left Bradycardia may be asymptomatic or associated with fatigue, Physicalifindings of bradycardia include hypotension, cool Bradycardia can be associated with a narrow QRS complex Narrow-complex bradycardia can be associated with a regular Wide-complex bradycardia can be associated with a regular Bradycardia in the adult is classically defined as heart rate between populations. ventricle per beat multiplied by heart rate. weakness, light-headedness, or syncope. extremit es, and cannon A waves (in the setting of AV dissociation). (<120 ms) or a wide QRS complex (>120 ms). rhythm or an irregular rhythm. rhythm or an irregular rhythm. 63 References 1. Mangrum JM, DiMarco JP. The evaluation and management of bradycardia. N Engl J Med. 2000;342(10):703-709. 2. Berne RML, Levy MN. Physiology. 4th ed. St. Louis, MO: Mosby Inc.; 1998. 3. hour continuous electrocardiographic monitoring in 50.male medical students without Brodsky M, Wu D, Denes P, Kanakis C, Rosen KM. Arrhythmias documented by 24 apparent heart disease. Am J Cardiol. 1977;39(3):390-395 4. Kantelip JP, Sage E, Duchene-Marullaz P. Findings on ambulatory electrocardiographic monitoring in subjects older than 80 years. Am J Cardiol. 1986;57(6):398-401. 5. Cunha BA. The diagnostic significance.of relative bradycardia in infectious disease. Clin Microbiol Infect. 2000;6(12):633-634 Page RL, Joglar JA, Caldwell MA, Calkins H, Conti JB, Deal BJ, et al. 2015 6. ACC/AHA/HRS guideline for the management of adult patients with supraventricular tachycardia: a report of the American College of Cardiology/American Heart Association Task Force on Clinical Practice Guidelines and the Heart Rhythm Society. Circulation. 2016;133(14):e506-e574. 7. Yasuma F, Hayano J. Respiratory sinus arrhythmia: why does the.heartbeat synchronize with respiratory rhythm? Chest. 2004;125(2):
683-690 Benditt DG, Gornick CC, Dunbar D, Almquist A, Pool-Schneider S. Indications for 8. electrophysiologic testing inIthe diagnosis and assessment of sinus node dysfunction. 9. Falk RH. Atrial fibrillation. N Engl J Med. 2001;344(14):1067-1078. Circulation. 1987;75(4 Pt 2):II 93-III102. 9. Levine HD, Smith C Jr. Repetitive paroxysmal tachycardia in adults. Cardiology. 1970;55(1):2-21. 10. Dreifus LS, Likoff W, eds. Cardiac Arrhythmias. New York: Grune and Stratton; 1973. 12. Merideth J, Pruitt RD. Cardiac arrhythmias. 5. Disturbances in cardiac conduction and their management. Circulation. 1973;47(5):
1098-1107. 11. Riera AR, Barros RB, de Sousa FD, Baranchuk A. Accelerated idioventricular rhythm: history and chronology of the main discoveries. Indian Pacing Electrophysiol J. 2010;10(1):40-48. 12. Vogler J, Breithardt G, Eckardt L. Bradyarrhythmias and conduction blocks. Rev Esp Cardiol (Engl Ed). 2012;65(7):656-667.

64

CHAPTER 2

65

Chest Pain

66 Case: A 76-year-old woman with an extra heart sound A 76-year-old woman with a history of hypertension and

hyperlipidemia presents to the clinic with increasingly frequent reports recenttepisodes of light-headedness on standing andrhas loste 2 years ago that showed no evidence of coronary artery disease episodes of chest pain over the course of a few months. The episodes occur several imes per day, lasting for up to 15 minutes. The pain is “squeezing” in quality. The episodes occur most frequently when sh walks around the grocery store. Rest alleviates the discomfo t. She consciousness on 2 occasions. She underwent coronary angiography (CAD). Blood pressure is 102/85 mm Hg. An extra heart sound is heard just obliterated by a grade III/VI late-peaking crescendo-decrescendo radiation to the carotids. The peripheral pulse is difficult to find and before S1 with the bell of the stethoscope over the apex; S2 is systolic murmur best heard over the right upper sternal border, with its contour is prolonged (slow rise and fall). Electrocardiogram (ECG) is shown in Figure 2-1.

FIGURE 2-1

What is the most likely cause of chest pain in this patient?

What are the 2 general sources of chest pain?

Is chest pain more often cardiac or noncardiac in nature?

Sources of chest pain can be cardiac or noncardiac.

Most cases of chest pain are noncardiac in nature.1

67

Cardiac Causes of Chest Pain

What are the 2 general subcategories of cardiac chest pain?

Cardiac chest pain can be related to acute coronary syndrome (ACS) or unrelated to ACS.

68

Acute Coronary Syndrome

What is acute coronary syndrome?

What are the characteristic symptoms of acute coronary syndrome? What differentiates acute myocardial infarction from myocardial ischemia? What are the 3 subtypes of acute coronary syndrome?

Why is it important to differentiate STEMI from UA/NSTEMI? What are the earliest electrocardiographic manifestations of STEMI? What electrocardiographic findings are diagnostic of STEMI?

What are the characteristic electrocardiographic findings of UA/NSTEMI?

Which groups of ECG leads are considered contiguous? What baseline conditions make electrocardiographic interpretation of ischemia

ACS describes a range of clinical syndromes that result in myocardial ischemia with or without infarction. The diagnosis relies on the identification of characteristic clinical features, including a compatible history, electrocardiographic findings, and biochemical markers.2 Patients with ACS often experience chest, upper extremity, mandibular, or epigastric discomfort that occurs with exertion or at rest, and typically lasts >20 minutes. These symptoms may be associated with diaphoresis, nausea, or syncope. Some patients experience “angina equivalent” symptoms such as dyspnea or fatigue. Symptoms of myocardial ischemia may be atypical in certain populations; particularly women, diabetics, and postoperative patients.3 Acute myocardial infarction is defined as myocardial necrosis that occurs as a result of myocardial ischemia, and can be identified by the rise and fall of cardiac biomarkers in the blood (eg, cardiac-specific troponin) along with other supportive evidence (eg, characteristic electrocardiographic changes).4

The subtypes of ACS are ST-elevation myocardial infarction (STEMI), unstable angina (UA), and non–ST-elevation myocardial infarction (NSTEMI).

STEMI and UA/NSTEMI are differentiated based on the presence of characteristic electrocardiographic findings. The distinction is important because acute treatment strategies differ between them. Immediate reperfusion therapy (eg, percutaneous coronary intervention or fibrinolytics) has been shown to be beneficial in patients with STEMI.2,3 An ECG should be obtained promptly for any patient suspected of having acute myocardial infarction. An early electrocardiographic manifestation of STEMI can be the presence of hyperacute (ie, broad-based, tall, symmetrical) T-waves in at least 2 contiguous leads (Figure 2-2). Transient Q waves may also be observed in acute myocardial ischemia.3 In the appropriate clinical context, the presence of new ST-segment elevation at the J point in 2 contiguous leads of ≥0.1 mV (except in leads V2-V3 where it must be ≥0.2 mV in men ≥40 years of age, ≥0.25 mV in men <40 years of age, or ≥0.15 mV in women) is diagnostic of STEMI (see Figure 2-2). It is important to recognize that ST-segment elevation can occur as a result of conditions other than myocardial infarction (eg, acute pericarditis).3

FIGURE 2-2 The evolution of ECG changes associated with STEMI. (From Lilly LS. Pathophysiology of Heart Disease: A Collaborative Projecttof Medical Students and Faculty. 6th ed. Philadelphia, PA: Wolters Kluwer Heal h; 2016.)

Electrocardiographic findings characteristic of UA/NSTEMI include new horizontal or downsloping ST-segment depression ≥0.05 mV in 2 contiguous leads and/or T-wave inversion ≥0.1 mV in 2 contiguous leads with prominent 
R wave or R/S ratio >1 (Figure 2-3).3

FIGURE 2-3 ECG abnormalities associated with UA/NSTEMI. (From Lilly LS. Pathophysiology of Heart Disease: A Collaborative Projecttof Medical Students and Faculty. 6th ed. Philadelphia, PA: Wolters Kluwer Heal h; 2016.)

Contiguous ECG leads include the anterior leads (V1-V6), inferior leads (II, III, aVF), and the lateral/apical leads (I, aVL). Additional leads such as V3R and V4R reflect the free wall of the right ventricle, and V7-V9 the 
infero-basal wall. ST-segment elevation or diagnostic Q waves in contiguous leads are more specific than ST-segment depression in localizing the region of myocardial ischemia or infarction.3 False-positive electrocardiographic features of ischemia may occur in the setting of early repolarization, left ventricular hypertrophy, left bundle branch block, ventricular paced rhythm, preexcitation, J-point elevation syndromes (eg, Brugada syndrome), acute pericarditis, myocarditis, subarachnoid hemorrhage, metabolic disturbances (eg, hyperkalemia), stress cardiomyopathy, and cholecystitis. False-negative findings may occur in the setting of prior myocardial infarction with Q waves and/or persistent ST-segment elevation, ventricular paced

69

unreliable? How are UA and NSTEMI distinguished?

rhythm, and left bundle branch block.3 UA and NSTEMI are subtypes of ACS, but can be differentiated based on the degree of mycoardial injury, reflected by the presence of serum biomarkers (eg, troponin). Unlike UA, NSTEMI is associated with elevated serum troponin. The diagnosis of UA relies mostly upon the clinical history; electrocardiographic features of ischemia may or may not be present.3

What are the causes of acute coronary syndrome?

The prototypical and most common mechanism of ACS. Rupture of an atherosclerotic plaque (Figure 2-4).3

FIGURE 2-4 Unstable atherosclerotic plaque with plaque

Consider this etiology in patients who recently underwent primary coronary intervention. A traveling clot. A 32-year-old man presents with crushing substernal chest pain, ST-segment elevation on ECG, positive serum biomarkers, and positive urine drug screen. Consider this etiology in pregnant women who present with ACS.

disruption and platelet aggregation in the acute coronary syndromes. (From Porth CM. Essentials of Pathophysiology: Concepts of Altered Health States. 4th ed. Philadelphia, PA: Wolters Kluwer; 2015.)

Stent thrombosis. Coronary artery embolism. Coronary artery vasospasm, including Prinzmetal’s (ie, variant) angina.

Coronary artery dissection.

What are the risk factors for coronary artery disease?

What criteria are used to establish a positive family history of premature coronary artery disease? How does the timing of stent thrombosis differ between bare metal stents and drug- eluting stents? What is the most common risk factor for coronary artery embolism? In patients with coronary artery vasospasm, what risk factor is associated with myocardial infarction (as opposed to myocardial ischemia)? What proportion of cases of coronary artery dissection occur in the peripartum period?

Risk factors for CAD include older age, male sex, smoking, hyperlipidemia, diabetes, physical inactivity, family history of premature CAD, and chronic inflammatory conditions.5 A family history of premature CAD is established when there is definite myocardial infarction or sudden death before 55 years of age in a first-degree male relative, or before 65 years of age in a first-degree female relative.5 Stent thrombosis is largely an early complication of bare metal stents (within 30 days); it tends to be a late complication of drug-eluting stents (after several years).6 Atrial fibrillation is the most common risk factor for coronary artery embolism.7

Coronary artery vasospasm is more likely to result in myocardial infarction in patients with underlying CAD.8 The majority of coronary artery dissections occur in women. Among them, one- third of cases occur in the peripartum period.9

70

Cardiac Causes of Chest Pain Unrelated to Acute Coronary Syndrome

What are the cardiac causes of chest pain Unrelated to Acute Coronary Syndrome?

A 62-year-old man complains of episodes of substernal chest pressure that reliably occur 10 minutes after he begins to mow the lawn, and resolve with rest. Pulsus parvus et tardus. Pleuritic chest pain that is relieved by leaning forward. “Ripping” chest pain associated with pulse discrepancies between extremities and dense mediastinum on chest radiography. Recurrent episodes of substernal chest pain at rest in the early morning hours in a patient with a negative cardiac stress test. Commonly caused by viral infections and may be associated with diffuse ST-segment elevation on ECG; some patients go on to develop dilated cardiomyopathy and chronic systolic heart failure. A 22-year-old man presents with several episodes of syncope and chest pain during exercise and is found to have a systolic ejection murmur on examination that intensifies with valsalva. A 26-year-old woman with an asthenic body type presents with recurrent episodes of anxiety, heart palpitations, and chest pain, and is found to have a midsystolic click followed by a murmur best heard over the apex of the heart.

What is the mechanism of stable angina?

Stable angina pectoris.

Aortic stenosis. Acute pericarditis. Aortic dissection.

Coronary artery vasospasm, including Prinzmetal’s angina.

Myocarditis.

Hypertrophic obstructive cardiomyopathy (HOCM).

Mitral valve prolapse (MVP).

Stable angina occurs when there is an obstructive but stable atherosclerotic plaque within a coronary artery that leads to relative myocardial hypoperfusion and ischemia in the territory it supplies when myocardial oxygen demand increases. In ambulatory patients, this can occur with light physical activity such as mowing the lawn. Hospitalized patients with stable angina may experience “supply/demand mismatch” when myocardial oxygen demand increases as a result of conditions such as tachycardia, hypotension, hypertension, congestive heart failure, anemia, hypoxemia, and sepsis (Figure 2-5).

71

Without treatment, what is the mean survival in patients with severe aortic stenosis associated with angina? What are the electrocardiographic findings of acute pericarditis? What risk factors are associated with acute aortic dissection?

What substances are associated with coronary artery vasospasm?

How often do patients with myocarditis present with chest pain? What is the expected change in quality of the murmur caused by hypertrophic obstructive cardiomyopathy when patients move from a standing to squatting position? Why do patients with symptomatic mitral valve prolapse often have persistent episodes of chest pain and palpitations after undergoing mitral valve replacement?

FIGURE 2-5 Stable fixed atherosclerotic plaque in stable angina. (From Porth CM. Essentials of Pathophysiology: Concepts of Altered Health States. 4th ed. Philadelphia, PA: Wolters Kluwer; 2015.)

Without treatment, mean survival in patients with severe aortic stenosis associated with angina is 5 years.10

The typical electrocardiographic findings of acute pericarditis include diffuse ST-segment elevation, diffuse PR-segment depression, and PR-segment elevation in lead aVR.11 Risk factors for acute aortic dissection include male sex, age in the 60s and 70s, hypertension, prior cardiac surgery (particularly aortic valve repair), bicuspid aortic valve, connective tissue diseases (eg, Marfan syndrome), and aortitis (eg, giant cell arteritis, syphilis).12,13 Coronary artery vasospasm can be provoked by a number of substances, including cocaine, amphetamines, marijuana, 5-fluorouracil, and sumatriptan. The cause of spontaneous coronary artery vasospasm (ie, Prinzmetal’s angina) has not been completely elucidated, but genetic and environmental factors may play a role. Active smokers constitute the majority of patients who suffer from spontaneous coronary artery vasospasm.14 Dyspnea is the most frequent symptom of acute or chronic myocarditis, reported by most patients on presentation. Around one-third of patients report chest pain on presentation.15 Moving from a standing to squatting position increases preload, which results in a decrease in the intensity of the murmur associated with HOCM.

Symptoms experienced by patients with MVP, including chest pain and palpitations, are often related to associated autonomic dysfunction that persists after the valvular lesion is addressed.16

72

Noncardiac Causes of Chest Pain

The causes of noncardiac chest pain can be separated into which system-based subcategories?

The causes of noncardiac chest pain can be separated into the following subcategories: pulmonary, Gastrointestinal, musculoskeletal, and other.

73

Pulmonary Causes of Chest Pain

What are the pulmonary causes of chest pain?

This condition is associated with an auscultatory sound akin to walking on fresh snow. Fever, purulent cough, and pleuritic chest pain. A 46-year-old woman from California with 
a history of ovarian cancer presents with sudden onset pleuritic chest pain after returning home from a trip to Thailand. Hyperresonance to percussion of the chest over the affected area. A previously healthy 36-year-old woman presents with dyspnea on exertion and is found to have elevated jugular venous pressure (JVP), a right ventricular heave, and a loud pulmonic component of the second heart sound (P2). A 24-year-old man from Saudi Arabia with a known blood disorder (he cannot recall the name) presents with diffuse arthralgias and sudden-onset chest pain.

What is pleurisy?

Does tactile fremitus increase or decrease over an area of pneumonia?

What are the electrocardiographic findings of acute pulmonary embolism?

What is the immediate treatment for tension pneumothorax? What is the definitive diagnostic study to evaluate for pulmonary hypertension?

In a patient with acute chest syndrome, what finding is often present on chest imaging?

Pleurisy. Pneumonia. Pulmonary embolism (PE).

Pneumothorax. Pulmonary hypertension.

Acute chest syndrome (related to sickle cell disease).

Pleurisy describes inflammation of the pleura and is associated with numerous conditions (eg, infection, medication, rheumatologic disease). The chest pain is typically pleuritic in nature, exacerbated by deep breathing, coughing, or sneezing. In addition to addressing the underlying cause, first-line pharmacologic treatment for the symptoms of pleurisy is nonsteroidal anti-inflammatory drugs.17 Tactile fremitus increases over an area of consolidation related to pneumonia. Other physical findings of consolidation include dullness to percussion, egophony (when the patient says the word “bee,” the “e” sounds more like an “a”), and whispered pectoriloquy (“chest-speaking”), which refers to an increase in volume and clarity of whispered sounds while listening to the chest with the stethoscope.18 Electrocardiographic manifestations of acute PE include sinus tachycardia, new atrial fibrillation, and evidence of right ventricular strain such as T-wave inversion in the anteroseptal leads, right axis deviation, new right bundle branch block, and S1Q3T3 pattern (S wave in lead I, Q wave in lead III, and inverted T wave in lead III). The ECG may be normal in up to one-quarter of patients.19 Tension pneumothorax (see Figure 24-3) should immediately be treated by placing a large-bore needle/catheter through the second anterior intercostal space. The catheter should remain in place until a thoracostomy tube can be inserted. History, physical examination, electrocardiography, and echocardiography can provide information suggestive of pulmonary hypertension. The gold standard diagnostic study to evaluate for pulmonary hypertension is right heart catheterization with direct measurement of pulmonary pressures. Consolidation is a common finding on chest imaging in patients with acute chest syndrome.

74

Gastrointestinal Causes of Chest Pain

What are the Gastrointestinal causes of chest pain?

A 48-year-old man with obesity presents to his primary care physician with complaints of chest pain and an acidic taste in his mouth after large meals. A 42-year-old woman who has been taking increasing doses of nonsteroidal anti-inflammatory drugs for chronic low back pain complains of episodes of epigastric and retrosternal chest discomfort after meals. Pain associated with this entity is typically located in the right upper quadrant of the abdomen but may radiate to the chest and right scapular region. Painful contractions of the esophagus. A 39-year-old man with a history of severe hypertriglyceridemia presents with epigastric and retrosternal chest pain that radiates to the back. A 36-year-old man with a history of heavy alcohol use is admitted for chest pain following an episode of violent retching and is found to have subcutaneous emphysema of the chest and a left-sided pleural effusion.

What lifestyle modifications are helpful in treating gastroesophageal reflux disease?

Which type of peptic ulcer disease often presents with pain that worsens with meals? How common is cholelithiasis?

How is the diagnosis of esophageal spasm made?

How is the diagnosis of acute pancreatitis made?

What is the most common cause of esophageal rupture?

Gastroesophageal reflux disease (GERD).

Peptic ulcer disease (PUD).

Biliary colic.

Esophageal spasm. Acute pancreatitis.

Esophageal rupture (Boerhaave syndrome).

Lifestyle modifications that may help treat GERD include avoidance of foods that are acidic or irritating (eg, citrus fruits, tomatoes, onions, spicy foods), avoidance of foods that cause gastric reflux (eg, fatty or fried foods, coffee, tea, caffeinated beverages, chocolate, mint), reduction of alcohol consumption, smoking cessation, and weight loss. Patients with postprandial symptoms may benefit from smaller and more frequent meals, and refraining from lying down after meals. For patients with nocturnal symptoms, food should not be consumed within 3 hours of bedtime, and the head of the bed should be elevated.20 Pain that worsens with meals is suggestive of gastric PUD. Pain that improves with meals is suggestive of duodenal PUD. In the industrialized world, 10% to 15% of adults develop gallstones. Asymptomatic patients develop symptoms at a rate of 1% to 4% per year. Risk factors for developing cholelithiasis include older age, female sex, obesity, family history, pregnancy, rapid weight loss, and parenteral nutrition.21 The diagnosis of esophageal spasm can be made with the combination of barium swallow and esophageal manometry testing. Pharmacologic treatment strategies include proton pump inhibitors, calcium channel blockers, nitrates, and phosphodiesterase inhibitors.22 The diagnosis of acute pancreatitis requires at least 2 of the following 3 features: (1) characteristic abdominal pain (eg, postprandial, epigastric, radiating to the back); (2) serum lipase or amylase levels at least 3 times the upper limit of normal; or (3) evidence of acute pancreatitis on cross-sectional imaging.23 The most common cause of esophageal rupture is medical instrumentation (eg, esophageal dilators for achalasia). Other causes are related to a variety of conditions that increase intraabdominal pressure; including intense vomiting or retching, weight lifting, parturition, and status epilepticus.24

75

76

Musculoskeletal Causes of Chest Pain

What are the musculoskeletal causes of chest pain?

These 3 related disorders typically respond well to nonsteroidal anti- inflammatory drugs. Often associated with neck pain. A 33-year-old man with a history of groin sarcoma presents with chest wall pain, and cross-sectional imaging of the chest reveals multiple nodules and masses throughout the mediastinum, pleurae, and osseous structures.

What physical finding is often present in patients with costochondritis, chest wall trauma, or rib fracture that can help differentiate these conditions from angina? What causes cervical angina?

What is the most common primary malignant tumor of the chest wall?

Costochondritis, chest wall trauma, and rib fracture. Radiculopathy (ie, cervical angina). Chest wall tumor.

Pain is often reproducible with palpation of the chest in patients with costochondritis, chest wall trauma, and rib fracture. The majority of cases of cervical angina are caused by cervical nerve root compression (C4-C8) associated with conditions such as degenerative disc disease.25 Soft tissue sarcoma is the most common primary malignant tumor of the chest wall. Other types of tumors include chondrosarcoma, osteosarcoma, small round cell tumor, plasmacytoma, and giant cell tumor.26

77

Other Causes of Chest Pain

What are the other causes of chest pain?

Patients with this condition may also suffer from Anxiety (panic disorder).1 agoraphobia, major depression, and substance abuse. During finals week of his first year in college, a previously Herpes zoster. healthy 18-year-old man presents to the clinic with pain over the right side of the chest associated with vesicular skin lesions on an erythematous base.

What is the treatment for panic disorder?

What is the typical distribution of herpes zoster?

The majority of patients with panic disorder (with or without agoraphobia) can be effectively treated with either nonpharmacologic modalities such as cognitive and behavioral techniques, or pharmacologic modalities such as selective serotonin reuptake inhibitors.1 The rash of herpes zoster tends to be asymmetric, following a unilateral dermatomal distribution.

78 Case Summary presents with escalating episodes oftexertional chest pain and is found A 76-year-old woman with hyper ension and hyperlipidemia to have narrow pulse pressure, a late-peaking systolic ejection

murmur, a weak and delayed peripheral pulse, and an abnormal ECG. What is the most likely cause of chest pain in this patient? Severe aortic stenosis.

79

Bonus Questions

How common is aortic stenosis?

the extra heart sound in this case? What is the most likely source of

electrocardiographic findings are What relevant present in this case? What physical findings correlate stenosis? with the severity of aortic What is the most likely cause of aortic stenosis in this case?

If the patient in this case were age be the most likely cause of aortic 46 instead of age 76, what would stenosis? What are the typical clinical

manifestations of aortic stenosis? What is the mechanism of chest stenosis? pain in patients with aortic How should patients with asymptomatic aortic stenosis be monitored? What is the treatment for

symptomatic severe aortic stenosis?

In the industrialized world,taortic stenosis occurs in 1% to 3% of the general population over 70 years of age. It is the most common cause of valvular location, pitch, and clinicaljhistory (see Figure 4-4). An S4 often occursjin the setting of concentric ventricular hypertrophy, an expectedlsequela ofon heart disease in this popula ion. 27 Extra heart sounds that occur near S1 include split S1, S4 gallop, and e ection clicks. The extra sound in this case is most likely an S4 gal op, based severe aortic stenosis. An e ection click might also be expected in the setting of aortic stenosis, but this sound is appreciated over the base of the heart. The ECG in this case (see Figure 2-1) demonstrates left ventricular hypertrophy and left axis deviation, findings that are compatible with the 28

underlying diagnosis of aortic stenosis. There are several different criteria used to diagnose left ventricular hypertrophy on ECG. In this case, the ECG meets the “R in aVL” score (R in aVL ≥11 mm), for example. 27,29 Physical findings that predict greater severity of aortic stenosis include a narrow pulse pressure, pulsus parvus et tardus (small, slowly rising and falling pulse), brachioradial pulse delay, diminished or inaudible aortic component of the second heart sound (A2), and a late-peaking systolic murmur. As the degree of stenosis worsens, the murmur peaks progressively later in systole. The intensity of the murmur does not correlate with severity. 28 In the industrialized world, age-related calcific degeneration of aortic leaflets—an inflammatory process similar to atherosclerosis—is the most common underlying cause of aortic stenosis in patients >70 years of age. Other causes include congenital disease (eg, bicuspid aortic valve) and rheumatic heart disease, but these conditions tend to present in younger patients.28 Congenital bicuspid aortic valve is the most common cause of aortic stenosis in younger patients. This is a condition in which the valve is composed of 2 leaflets instead of 3. It affects 1% to 2% of the general population in the industrialized world. Because of abnormal hemodynamics across the bicuspid valve, these patients develop degenerative changes at an earlier age.27 The 3 classic manifestations of aortic stenosis are angina, presyncope/syncope, and dyspneairelated to heart failure; symptoms typically develop ins that order as the disease progresses. When untreated, mean survival is 5 years in patients w th severe aortic stenosis associated with angina, 3 year in patients with syncope, and 2 years in patients with heart failure. 10,27 Severe aortic stenosis results in compensatory ventricular hypertrophy as a means to maintaintcardiac output. This leads to Decreased compliance and diastolic dysfunction. Eventually, the hypertrophied myocardium is unable to keep up wi h increasing oxygen demands, resulting in myocardial ischemia. 27 Patients with asymptomatic aortic stenosis and preserved cardiac function should be monitored with serial echocardiography (every 5 years for mild disease, every 3 years for moderate disease, and annually for severe disease). 27 The definitive treatment for symptomatic severe aortic stenosis is aortic valve replacement. Options include conventional aortic valve replacement (with either a mechanical or bioprosthetic valve) or transcatheter aortic valve replacement. 27

80 Key Points

The majority oftcases of chest pain are noncardiac in nature. Sources of ches pain can be cardiac or noncardiac. Cardiac chest pain can be related to ACS or unrelated to ACS. The symptoms of ACS include chest, upper extremity, mandibu

or epigastric discomfort that occurs with exertion or atirest, andlar, ACS includes the following distinct clinical entities: STEMI, UA, The electrocardiographic features of STEMI include hyperacute T typically lasts >20 minutes; associated symptoms may nclude diaphoresis, nausea, or syncope. and NSTEMI. waves and ST-segment elevation in at least 2 contiguous leads. The electrocardiographic features of UA/NSTEMI include ST-

segment depression and T-wave inversion in at least 
2 contiguous STEMI and NSTEMI are associated with elevated serum leads. biomarkers. Acute plaque rupture is the prototypical and most common The causes of noncardiac chest pain can be separated into the Gastrointestinal, musculoskeletal, and other. mechanism of ACS. following system-based subcategories: pulmonary,

81

References 1. Fleet RP, Beitman BD. Unexplained chest pain: when is it panic disorder? Clin Cardiol. 1997;20(3):187-194.

  1. Smith JN, Negrelli JM, Manek MB, Hawes EM, Viera AJ. Diagnosis and management of acute coronary syndrome: an evidence-based update. J Am Board Fam Med.

2015;28(2):283-293. 3. Thygesen K,iAlpert JS, Jaffe AS, et al. Third universal definition of myocardial infarction. C rculation. 2012;126(16):2020-2035. Alpert JS, Thygesen K, Antman E, Bassand JP. Myocardial infarction redefined—a consensus document of The Joint European Society of Cardiology/American College o 4. Cardiology Committee for the redefinition of myocardial infarction. J Am Coll Cardiol. f 2000;36(3):959-969.

  1. National Cholesterol Education Program Expert Panel on Detection, Evaluation, and Cholesterol Education Program (NCEP) Expert Panel on Detection, Evaluation, and Treatment of High Blood Cholesterol in Adults. Third report of the National Treatment of High Blood Cholesterol in Adults (Adult Treatment Panel III) final report. Circulation. 2002;106(25):3143-3421.
  2. Kirtane AJ, Stone GW. How to minimize stent thrombosis. Circulation. 2011;124(11):1283-1287.

  3. Shibata T, Kawakami S, Noguchi T, et al. Prevalence, clinical features, and prognosis of acute myocardial infarction attributable to coronary artery embolism. Circulation.

2015;132(4):241-250. 8. prognosis of patients with variant angina. Circulation. 1987;76(5):990-997.term Walling A, Waters DD, Miller DD, Roy D, Pelletier GB, Theroux P. Long- 9. DeMaio SJ Jr, Kinsella SH, Silverman ME. Clinical course and long-term.prognosis of 10. Ross J Jr, Braunwald E. Aortic stenosis. Circulation. 1968;38(1 suppl):61-67. spontaneous coronary artery dissection. Am J Cardiol. 1989;64(8):471-474 10. Spodick DH. The.Pericardium: A Comprehensive Textbook. New York, NY: Marcel Dekker, Inc.; 1997 11. Criado FJ. Aortic dissection: a 250-year perspective. Tex Heart Inst J. 2011;38(6):694- 700.

  1. Nienaber CA, Clough RE. Management of acute aortic dissection. Lancet. 2015;385(9970):800-811.

  2. Lanza GA, Careri G, Crea F. Mechanisms of coronary artery spasm. Circulation. 2011;124(16):1774-1782.

  3. Hufnagel G, Pankuweit S, Richter A,iSchonian U, Maisch B. The European Study of Epidemiology and Treatment of Card ac Inflammatory Diseases (ESETCID). First epidemiological results. Herz. 2000;25(3):279-285.

  4. GaffneylFA, Karlsson ES, Campbell W, et al. Autonomic dysfunction in women with mitral va ve prolapse syndrome. Circulation. 1979;59(5):894-901.

  5. Kass SM, Williams PM, Reamy BV. Pleurisy. Am Fam Physician. 2007;75(9):1357-1364. 18. Sapira JD. The Art & Science of Bedside Diagnosis. Baltimore, MD: Urban & Schwarzenberg Inc.; 1990. . Sreeram N, Cheriex EC, Smeets JL, Gorgels AP, Wellens HJ. Value of the 12-lead

19electrocardiogram at hospital admission in the diagnosis of pulmonary embolism. Am J Cardiol. 1994;73(4):298-303.

82

  1. Kahrilas PJ. Clinical practice. Gastroesophageal reflux disease. N Engl J Med. 2008;359(16):1700-1707.

  2. Sanders G, Kingsnorth AN. Gallstones. BMJ. 2007;335(7614):295-299.

  3. Tutuian R, Castell DO. Review article: oesophageal spasm—diagnosis and management. Aliment Pharmacol Ther. 2006;23(10):1393-1402.

  4. Forsmark CE, Vege SS, Wilcox CM. Acute pancreatitis. N Engl J Med. 2016;375(20):1972-1981.

  5. Soreide JA, VistetA. Esophageal perforation: diagnostic work-up and clinical decision-25. Sussman WI, Makovitch SA,iMerchant SH, Phadke J. Cervical angina: an overlooked making in the firs 24 hours. Scand J Trauma Resusc Emerg Med. 2011;19:66. source of noncardiac chest pa n. Neurohospitalist. 2015;5(1):22-27.

  6. Bagheri R, Haghi SZ,.Kalantari MR, et al. Primary malignant chest wall tumors: analysis of 40 patients J Cardiothorac Surg. 2014;9:106.
  7. Zakkar M, Bryan AJ, Angelini GD. Aortic stenosis: diagnosis and management. BMJ. 2016;355:i5425.

  8. Marriott HJL. Bedside Cardiac Diagnosis. Philadelphia, PA: Lippincott Company; 1993. 29ventricular,hypertrophy in general population. Eur J Epidemiol. 2008;23(4):261-271. . Casiglia E Schiavon L, Tikhonoff V, et al. Electrocardiographic criteria of left

83

CHAPTER 3

84

Heart Block

85 Case: An 82-year-old man with an abnormal jugular venous waveform An 82-year-old man with a history of coronary artery disease, hypertension, and hyperlipidemia is admitted o the hospital with

episodes of light-headedness over theipast fewtdays. Symptoms occur at rest and with activity and are assoc ated with palpitations. Heart rate is 48 beats per minute, and blood pressure is

140/40 mm Hg. Jugular venous pressure is estimated to be 8 cm H O. 2 Qualitative analysis of the jugular venous waveform reveals a large el Laboratory studies are unremarkable. The rhythm strip of the outward pulsation that occurs intermittently. ectrocardiogram (ECG) is shown in Figure 3-1.

FIGURE 3-1

What is the most likely cause of light-headedness in this patient?

What is heart block? What is the path of electrical conduction in the normal heart? How is heart rate regulated? What is the relationship between cardiac output and heart rate? What are the symptoms of heart block? What are the physical findings of heart block? What are the 3 general types of heart block?

Heart block refers to disruption of electrical conduction that normally occurs sequentially from the atria to the ventricles. In the normal heart, an impulse spontaneously originates from the sinoatrial (SA) node, which is located in the subepicardial surface at the junction of the right atrium and superior vena cava. The impulse propagates through the myocytes of the right and left atria simultaneously before reaching the Atrioventricular (AV) node, which is located in the inferior portion of the right atrium. From there, the impulse is conducted to the bundle of His within the membranous septum, which then separates into the right and left bundle branches supplying the right and left ventricles, respectively (see Figure 1-2).

The sympathetic and parasympathetic nervous systems innervate the conduction system of the heart. Parasympathetic tone decreases SA node automaticity and AV node conduction, whereas sympathetic input increases SA node automaticity and AV node conduction.1 Cardiac output (CO) is equal to the forward stroke volume (SV) of the left ventricle per beat multiplied by heart rate (HR).1CO =SV ×HR

Many patients with heart block are asymptomatic. Symptoms depend on the type of heart block; however in general, may include fatigue, dyspnea, weakness, light-headedness, and syncope.2

Physical findings of heart block may include hypotension, cool extremities, and qualitative changes in the jugular venous waveform (eg, cannon A waves if there is AV dissociation).

The 3 general types of heart block are first-degree AV block, second-degree AV block, and third-degree AV block (ie, complete heart block).

86

87

First-Degree Atrioventricular Block

What electrocardiographic finding is diagnostic of first- degree AV block?

What does the PR interval measure? What structure regulates conduction between the atria and the ventricles? Which main coronary artery supplies the AV node in most patients? Is first-degree AV block associated with nonconducted (or dropped) beats? How common is first-degree AV block? What are the symptoms of first- degree AV block? What are the acquired causes of first-degree AV block?

What is the prognosis of first- degree AV block? What is the treatment for first-degree AV block that is unrelated to a reversible cause?

First-degree AV block is defined by a PR interval >200 ms on ECG (Figure 3-2).3

FIGURE 3-2 First-degree AV block. The PR interval is >200 ms in duration. From Lilly LS. Pathophysiology of Heart Disease: A Collaborative Project of Medical Students and Faculty. 6th ed. Philadelphia, PA: Wolters Kluwer Health; 2016.)

The PR interval measures the time between the onset of atrial depolarization and the onset of ventricular depolarization (see Figure 1-2). The AV node regulates conduction between the atria and the ventricles (see Figure 1-2).

The AV nodal artery originates from the right coronary artery in 80% of patients and the circumflex in 10%; it arises from both in 10% of patients.1

First-degree AV block is not associated with nonconducted beats. The term is somewhat of a misnomer because there is only delayed AV conduction without actual block.4

In the industrialized world, the prevalence of first-degree AV block in the general population is approximately 1% in individuals <60 years of age, and 6% in individuals >60 years of age. Nonmodifiable risk factors include male sex, increasing age, and genetic factors.5,6 First-degree AV block is generally asymptomatic. However, more severe cases (ie, PR interval >300 ms) can be associated with symptoms of dyspnea and light-headedness that usually worsen with exercise as a result of the loss of AV synchrony.2,4 Acquired causes of first-degree AV block include idiopathic progressive degeneration of the cardiac conduction system (ie, Lenègre’s disease and Lev’s disease), medications (eg, β-blockers), procedures (eg, postcatheter ablation), enhanced vagal tone (eg, athletes), electrolyte disturbances (eg, hypokalemia), myocardial ischemia (most commonly inferior territory), endocarditis, myocarditis, infections (eg, Lyme disease), certain muscular dystrophies (eg, myotonic muscular dystrophy), and infiltrative diseases (eg, amyloidosis). Any reversible secondary causes that are identified should be addressed.2,3,6 Generally, first-degree AV block is a benign condition with an excellent prognosis. However, there is some evidence that these patients are at slightly Increased risk of developing more serious conduction abnormalities (eg, atrial fibrillation) and all-cause mortality.4,7 Most patients with first-degree AV block are asymptomatic and prognosis is excellent without treatment. However, in patients with symptoms related to severe PR prolongation (>300 ms), implantation of a pacemaker may be considered (although there is no evidence that pacemakers improve survival in this setting).2,4

88

Second-Degree Atrioventricular Block

What are the 2 general types of second- degree AV block?

The 2 general types of second-degree AV block are Mobitz type I (Wenckebach) and Mobitz type II.

89

Mobitz I Second-Degree Atrioventricular Block

What electrocardiographic findings are diagnostic of Mobitz I second- degree AV block?

Second-degree AV block is characterized by the presence of both conducted beats (ie, P wave followed by an associated QRS complex) and nonconducted (or dropped) beats (ie, P wave not followed by an associated QRS complex). Mobitz I is defined by the presence of nonconducted beats that are preceded by conducted beats associated with progressively longer PR intervals on ECG. This is most easily appreciated by measuring the PR intervals before and after the nonconducted beat. The PR interval immediately after the nonconducted P wave returns to its baseline value and is shorter than the PR interval before the nonconducted beat (Figure 3-3). Mobitz I was originally described by Wenckebach using tracings of the jugular venous waveform. He observed A-C prolongation leading up to dropped beats.3

FIGURE 3-3 Mobitz I second-degree AV block. The P-wave rate is constant, but the PR interval progressively lengthens

What is the typical location of block within the conduction system in Mobitz I second- degree AV block? Mobitz I second-degree AV block is most commonly associated with myocardial infarction involving which vascular territory? What are the symptoms of Mobitz I second- degree AV block? What are the acquired causes of Mobitz I second- degree AV block? What is the prognosis of Mobitz I second- degree AV block? What is the treatment for Mobitz I second-degree AV block that is unrelated to a reversible cause?

until a QRS is completely blocked (after the fourth P wave). From Lilly LS. Pathophysiology of Heart Disease: A Collaborative Project of Medical Students and Faculty. 6th ed. Philadelphia, PA: Wolters Kluwer Health; 2016.)

The typical location of block in Mobitz I is the AV node.8

Mobitz I is most commonly associated with myocardial infarction involving the distribution of the right coronary artery, which supplies the AV node in most patients. Look for corresponding ST-segment elevation in the inferior leads (II, III, aVF) accompanying the rhythm disturbance.9

Mobitz I is generally asymptomatic. Symptoms such as dyspnea, palpitations, and light-headedness are rare.3,8

The acquired causes of Mobitz I are similar to those of first-degree AV block. Any reversible secondary causes that are identified should be addressed.2,3,6

Mobitz I is generally associated with a good prognosis; progression to higher degrees of AV block is uncommon. However, factors such an infranodal location of block may be associated with higher risk.2,3

Treatment for Mobitz I is usually unnecessary in asymptomatic patients. In rare cases when patients are symptomatic and hemodynamically unstable, urgent pharmacologic treatment (eg, atropine) or temporary cardiac pacing should be pursued. Placement of a permanent pacemaker may be necessary in cases of symptomatic Mobitz I.2,3

90

Mobitz II Second-Degree Atrioventricular Block

What electrocardiographic findings are diagnostic of Mobitz II second-degree AV block?

Mobitz II is characterized by the presence of conducted beats (with a constant PR interval) followed by sudden failure of P wave conduction (ie, a nonconducted beat) (Figure 3-4).3

FIGURE 3-4 Mobitz II second-degree AV block. A QRS complex is blocked (after the fourth P

What is the typical location of block within the conduction system in Mobitz II second- degree AV block? Mobitz II second-degree AV block is most commonly associated with myocardial infarction involving which vascular territory? What are the symptoms of Mobitz II second- degree AV block? What are the acquired causes of Mobitz II second-degree AV block?

What is the prognosis of Mobitz II second- degree AV block? What is the treatment for Mobitz II second- degree AV block that is unrelated to a reversible cause?

What ratio of AV conduction makes the electrocardiographic distinction between Mobitz I and Mobitz II second-degree AV blocks challenging?

wave) without gradual lengthening of the preceding PR intervals. Although the QRS width in this example is narrow, it is often widened in patients with Mobitz II. From Lilly LS. Pathophysiology of Heart Disease: A Collaborative Project of Medical Students and Faculty. 6th ed. Philadelphia, PA: Wolters Kluwer Health; 2016.)

The typical location of block in Mobitz II is below the AV node, within the His-Purkinje system or bundle branches.3,8 Mobitz II is most commonly associated with myocardial infarction involving the distribution of the left anterior descending artery, which supplies the His-Purkinje system and bundle branches in most patients. Look for corresponding ST-segment elevation in the anterior leads (V1-V4) accompanying the rhythm disturbance.9 Patients with Mobitz II are frequently symptomatic and may complain of dyspnea, palpitations, light-headedness, and syncope.3,8 The acquired causes of Mobitz II are similar to those of first-degree AV block and Mobitz I second-degree AV block. Any reversible secondary causes that are identified should be addressed.2 Mobitz II is associated with a high rate of progression to third-degree AV block, and Increased mortality.2,3 Treatment for Mobitz II is virtually always necessary. In cases when patients are symptomatic and hemodynamically unstable, urgent pharmacologic treatment (eg, dopamine) or temporary cardiac pacing should be pursued. Given its unstable nature, placement of a permanent pacemaker is typically necessary in patients with Mobitz II.2,3 It is difficult to distinguish between Mobitz I and Mobitz II electrocardiographically in the presence of second-degree 2:1 AV block.

91

Second-Degree 2:1 Atrioventricular Block

What are the electrocardiographic findings of second-degree 2:1 AV block?

Second-degree 2:1 AV block is defined by a pattern of alternating conducted and nonconducted beats (Figure 3-5). This pattern makes it impossible to define the underlying rhythm as Mobitz I or Mobitz II electrocardiographically because an assessment of sequential PR intervals cannot be made. However, it is important to determine the level of block (ie, nodal or infranodal) because of prognostic and therapeutic implications. Infranodal block carries a poorer prognosis, and placement of a permanent pacemaker is indicated.1,2

FIGURE 3-5 Second-degree AV block with 2:1 AV conduction. There are twice as many P waves (P) as QRS

When there is 2:1 AV block, what electrocardiographic features can help determine the level of AV block? When there is 2:1 AV block, what maneuvers can be performed to help determine the level of AV block?

complexes, indicating that every other atrial impulse is blocked. From Katz AM. Physiology of the Heart. 5th ed. Philadelphia, PA: Lippincott Williams & Wilkins; 2011.)

In the setting of 2:1 AV block, several electrocardiographic features can help distinguish the level of block: (1) When 2:1 AV block is associated with a narrow QRS complex, it is likely that the level of block is in the AV node, whereas a wide QRS suggests that the level of block is infranodal. (2) Fixed 2:1 AV block with a PR interval longer than 280 ms suggests block at the level of the AV node, whereas a PR interval shorter than 160 ms suggests infranodal block. (3) Presence of Mobitz I AV block before or after episodes of 2:1 AV block is highly suggestive of block at the level of the AV node.2,3,10 In the setting of 2:1 AV block, maneuvers that increase heart rate and AV conduction (eg, exercise) typically improve conduction when the level of block is nodal (eg, the ratio of AV conduction may improve from 2:1 to 1:1), but worsen conduction when the level of block is infranodal (eg, the ratio of AV conduction may worsen from 2:1 to 3:1). Maneuvers that decrease heart rate and AV conduction (eg, carotid massage) typically worsen conduction when the level of block is nodal, but improve conduction when the level of block is infranodal.2,3

92

Third-Degree Atrioventricular Block

What electrocardiographic findings are diagnostic of third- degree AV block?

Third-degree AV block is defined by a total lack of AV conduction. This is characterized electrocardiographically by the presence of regular P-P and R-R intervals but with complete dissociation of P waves and QRS complexes (usually atrial rate >ventricular rate) (Figure 3-6).

FIGURE 3-6 Third-degree AV block. The P wave and QRS rhythms are independent of one another. The QRS

Is the QRS complex narrow or wide in the setting of third-degree AV block? What are the symptoms of third- degree AV block? What are the acquired causes of third-degree AV block? What is the prognosis of third-degree AV block? What is the treatment for third-degree AV block that is unrelated to a reversible cause?

complexes are widened because they originate within the distal ventricular conduction system, not at the bundle of His. The second and fourth P waves are superimposed on normal T waves. From Lilly LS. Pathophysiology of Heart Disease: A Collaborative Project of Medical Students and Faculty. 6th ed. Philadelphia, PA: Wolters Kluwer Health; 2016.)

Third-degree AV block can produce either a narrow or wide QRS complex, depending on the location of the escape rhythm. If the escape rhythm originates above the level of the bundle of His, the QRS complex will usually be narrow (with a rate between 40-60 beats per minute); if it is generated below the level of the bundle of His, the QRS complex will be wide (with a rate between 20-40 beats per minute).2,3 Patients with third-degree AV block are usually symptomatic, and may complain of dyspnea, palpitations, light- headedness, and syncope.3,8 The acquired causes of third-degree AV block are similar to those of first-degree and second-degree AV block. Any reversible secondary causes that are identified should be addressed.2,3 The prognosis of third-degree AV block is generally poor, particularly when patients are symptomatic. In patients with syncope related to third-degree AV block, the 1-year mortality rate can be as high as 50%.2,3 Treatment for third-degree AV block is virtually always necessary. In cases when patients are symptomatic and hemodynamically unstable, urgent pharmacologic treatment (eg, dopamine) or temporary cardiac pacing should be pursued. Given its unstable nature, placement of a permanent pacemaker is typically necessary in patients with third-degree AV block.2,3

93 Case Summary is An 82-year-old man presentsiwith episodes of light-headedness and What is the most likely cause of light-headedness in this patient? Third-degree AV block. found to have bradycardia, wide pulse pressure, and large intermittent venous pulsations n the neck.

94

Bonus Questions

What are the relevant electrocardiographic The ECG in this case (see Figure 3-1),demonstrates wide-complex bradycardia with regular P-P and R-R intervals. The P waves are findings in this case? dissociated from the QRS complexes consistent with third-degree AV block. What is the significance of the blood Wide pulse pressure can be a manifestation of third-degree AV block. Other hemodynamic consequences of third-degree AV block include pressure in this case? elevated right-sided cardiac pressures, Increased systemic and pulmonary vascular resistance, and reduced cardiac output despite Increased stroke volume.11 What abnormality of the jugular venous The large intermittent venous pulsations described in this case are most likely cannon A waves. waveform is described in this case? Why is third-degree AV block associated The A wave of the jugular venous waveformioccurs asia result of rightlatrial contraction. In the setting.of third-degree AV block, when the with cannon A waves? there is Atrioventricular dyssynchrony, the r ght atrium intermittently contracts against a closed tricuspid valve, producing a spike in pressure within the right atrium, which is then transm tted to the jugu ar vein as a large positive wave For a video of cannon A waves, see associated reference.12 Is the escape rhythm in this case more likely The ECG in this case (see Figure 3-1) demonstrates a wide QRS complex, indicative of a focus below the bundle of His. Notably, the heart arising from above or below the bundle of rate is slightly higher than expected for a ventricular escape rhythm. His? What is the most likely cause of third- The most common cause of third-degree AV block is idiopathic progressive degeneration of the cardiac conduction system (ie, Lenègre’s degree AV block in this case? disease and Lev’s disease), which is indeed the most likely cause in this case. A thorough investigation into other potentially reversible secondary causes should be pursued.2,3 What long-term treatment strategy should If no reversible cause of third-degree AV block is identified, the patient in this case would benefit from placement of a permanent be offered to the patient in this case if no pacemaker. 2,3 reversible cause of third-degree AV block is identified?

95 Key Points

Heart block can be asymptomatic or associated with fatigue, Physicalifindings of heart block include hypotension, cool The 3 general types of heart block are first-degree AV block, Second-degree AV block can be subdivided into Mobitz I and weakness, dyspnea, light-headedness, or syncope. extremit es, and cannon A waves (in the setting of AV dissociation). second-degree AV block, and third-degree AV block. Mobitz II. Electrocardiography is the simplest method to distinguish between First-degree heart block is defined by a PR interval >200 ms. electrocardiographically by nonconducted beats preceded by the various types of AV block. Mobitz I second-degree AV block is defined conducted beats with progressively longer PR intervals. Mobitz II second-degree AV block is defined

electrocardiographically by the presence of conducted beats with a constant PR interval, followed by sudden failure of P wave conduction (ie, nonconducted beats).

Second-degree 2:1 AV block is defined electrocardiographically by imperative to determine the level of block (ie, nodal or infranodal) a pattern of alternating conducted and nonconducted beats. It is because of therapeutic implications. Third-degree AV block is defined electrocardiographically by

regular P-P and R-R intervals, but with complete dissociation 
of P There are a variety of reversible causes of each type of AV block. waves and QRS complexes. Hemodynamically unstable AV block requires acute treatment with medications (eg, atropine) and/or temporary pacing.

AV block, particularly when it is of higher grade, often requires treatment with placement of a permanent pacemaker.

96

References 1. Mangrum JM, DiMarco JP. The evaluation and management of bradycardia. N Engl J Med. 2000;342(10):703-709.

  1. Vogler J, Breithardt G, Eckardt L. Bradyarrhythmias and conduction blocks. Rev Esp Cardiol. 2012;65(7):656-667.

  2. Merideth J, Pruitt RD. Cardiac arrhythmias. 5. Disturbances in cardiac conduction and their management. Circulation. 1973;47(5):1098-1107.

  3. Holmqvist F, Daubert JP. First-degree AV block-an entirely benign finding or a potentially curable cause of cardiac disease? Ann Noninvasive Electrocardiol.

2013;18(3):215-224. 5. and adverse cardiovascular outcomes: a systematic review and meta-analysis. Heart. Kwok CS, Rashid M, Beynon R, et al. Prolonged PR interval, first-degree heart block 2016;102(9):672-680.

  1. Nikolaidou T, Ghosh JM, Clark AL. Outcomes related to first-degree Atrioventricular block and therapeutic implications in patients with heart failure. JACC Clin

Electrophysiol. 2016;2(2):181-192. 7. prolonged PR interval or first-degree Atrioventricular block. JAMA. 2009;301(24):2571- Cheng S, Keyes MJ, Larson MG, et al. Long-term outcomes in individuals with 2577. Dhingra RC, Denes P, Wu D, Chuquimia R, Rosen KM. The significance of second

  1. degree Atrioventricularlblock and bundle branch block. Observations regarding site 9. Langendorf R, Pick A. Atrioventricular block, type II (Mobitz)–its nature and clinical and type of block. Circu ation. 1974;49(4):638-646. significance. Circulation. 1968;38(5):819-821.

  2. Josephson ME. Clinical Cardiac Electrophysiology:;Techniques and Interpretations. 4th ed. 11. Stack MF, Rader B, Sobol BJ, Farber SJ, Eichna LW. Cardiovascular hemodynamic Philadelphia, PA: Lippincott Williams & Wilkins 2008. functions in complete heart block and the effect of isopropylnorepinephrine.

Circulation. 1958;17(4, Part 1):526-536. 12. Tung MK, Healy S. Images in clinical medicine. Cannon A waves. N Engl J Med. 2016;374(4):e4.

97

CHAPTER 4

98

Heart Failure

99 Case: A 66-year-old woman with orthostatic hypotension syA 66-year-old woman with a history of hypertension, carpal tunnel with progressive dyspnea on exertion, orthopnea, and paroxysmal biventricular concentric hypertrophy with normal systoliclfunction. d ndrome, and heart failure of unknown etiology is referred to cardiology for evaluation. Her symptoms first began 6 months ago nocturnal dyspnea. At that time, echocardiography revealed She was diagnosed with congestive heart failure thought to be relate to hypertension, and has since been treated symptomatica ly with diuretics. Blood pressure has been well controlled with antihypertensive medications. Progressive symptoms and recent

episodes of syncope prompted referral to cardiology for evaluation. blood pressure is 118/84 mm Hg. On standing, heart rate is 89 beats In the recumbent position, heart rate is 90 beats per minute and per minute and blood pressure is 92/60 mm Hg. Jugular venous pressure (JVP) is 16 cm H O. An extra heart sound is heard just before 2 S1 with the bell of the stethoscope over the apex. 1. Electrocardiogram (ECG) is shown in Figure 4-

FIGURE 4-1 (From Moscucci M. Grossman & Baim’s Cardiac Catheterization, Angiography, and Intervention. 8th ed. Philadelphia, PA: Lippincott Williams & Wilkins; 2014.)

concentric hypertrophy with preserved systolic function. Cardiac diastolic filling, normal systolic function, and diffuse biventricularrast demonstrates extracellular amorphous hyaline deposits that turn an Repeat echocardiography shows progressive biventricular magnetic resonance imaging (MRI) demonstrates restriction of wall thickening with heterogeneous enhancement on delayed cont exposure. Endomyocardial biopsy with Congo red staining apple-green color under polarized light. What is the most likely cause of heart failure in this patient?

100

What is heart failure? Heart failure is a clinical syndrome that develops as a result of structural or functional impairment of ventricular filling or the ejection of blood.1 What modifiable risk Heart failure occurs more frequently in patients with hypertension, diabetes mellitus, metabolic syndrome, and factors are associated atherosclerotic disease.1 with heart failure? How common is heart In the industrialized world, heart failure is estimated to affect 2% of individuals 65 to 69 years of age, and 8% of failure? individuals ≥80 years of age. Black patients are disproportionately affected.1 What are the symptoms Symptoms of heart failure may include dyspnea, cough, orthopnea, paroxysmal nocturnal dyspnea, fatigue or of heart failure? lethargy, weight gain, light-headedness, nausea, early satiety, and abdominal discomfort.1 What are the physical Physical findings of right-sided heart failure may include tachycardia, hypotension, elevated JVP, right findings of right-sided ventricular heave, right-sided gallop (heard best at the left lower sternal border), ascites, and lower extremity heart failure? edema.1 What are the physical Physical findings of left-sided heart failure may include tachycardia, hypotension, narrow pulse pressure, end-findings of left-sided inspiratory crepitant rales on auscultation of the lungs, diffuse expiratory wheeze (ie, cardiac asthma), left-sided heart failure? gallop (heard best at the apex), laterally displaced apical impulse, pulsus alternans (in end-stage disease), and cool extremities (in cardiogenic shock).1 Are inspiratory rales In patients with chronic left-sided heart failure, the lungs may be clear as a result of adaptive dilation of the always present in pulmonary lymphatic vessels, which prevents the development of pulmonary edema despite the presence of an patients with left-sided elevated wedge pressure and pulmonary congestion.1 heart failure? What is the prognosis The prognosis of heart failure depends on patient-specific factors and the underlying cause of heart failure but, of heart failure? overall, half of patients die within 5 years of diagnosis.1 What are the 2 general Heart failure can be associated with reduced left ventricular systolic function (ie, systolic dysfunction) or categories of heart preserved left ventricular systolic function (ie, diastolic dysfunction). failure based on left ntricular fu ction? ve Otherncommon categorizations of heart failure include right-sided or left- sided heart failure, dilated or restrictive cardiomyopathy, and ischemic or nonischemic cardiomyopathy.

What is the definition of heart failure with Heart failure with preserved systolic function is defined as the presence of the clinical preserved systolic function? syndrome of heart failure with normal or near-normal left ventricular ejection fraction (>50%).2 Is heart failure more often associated with Patients with heart failure are equally divided between those with reduced systolic reduced systolic function or preserved systolic function and those with preserved systolic function.2 funIt is important to note that there is considerable overlap between these ction? categories of heart failure. In patients with reduced systolic function, there is often concomitant diastolic dysfunction. Furthermore, many diseases that are classified under heart failure with preserved systolic function can and often do eventually lead to heart failure with reduced systolic function.

101

Heart Failure With Reduced Left Ventricular Systolic Function

What type of myocardial hypertrophy is typically associated with heart failure with reduced systolic function?

Heart failure with reduced systolic function is associated with eccentric hypertrophy. The chambers of the heart dilate and the myocardial walls thin (Figure 4-2).

FIGURE 4-2 Different types of myocardial hypertrophy. A, Normal symmetric hypertrophy with proportionate

What is the manifestation of eccentric hypertrophy on chest radiography?

Is heart failure with reduced systolic function typically associated with dilated or restrictive cardiomyopathy? What extra heart sound is commonly associated with heart failure with reduced systolic function?

increases in myocardial wall thickness and length. B, Concentric hypertrophy with a disproportionate increase in wall thickness, resulting in a decrease in chamber size (arrow). C, Eccentric hypertrophy with ventricular dilation and a decrease in wall thickness (curved arrow), resulting in an increase in chamber size. (From Porth CM. Essentials of Pathophysiology Concepts of Altered Health States. 2nd ed. Philadelphia: Lippincott Williams & Wilkins; 2007.)

Eccentric hypertrophy manifests as an enlarged cardiac silhouette on chest radiography. In adults, an enlarged cardiac silhouette is generally defined by a cardiothoracic ratio ≥0.5. The cardiothoracic ratio is measured by dividing the transverse diameter of the heart by the maximum internal diameter of the thoracic cavity. Be careful not to diagnose “cardiomegaly” on the chest radiograph, as there are other conditions that can cause an enlarged cardiac silhouette (eg, pericardial effusion).3 Heart failure with reduced systolic function is typically associated with dilated cardiomyopathy. Eccentric hypertrophy results in thinned ventricular myocardium with reduced contractility (ie, the heart becomes “big and floppy”).

An S3 gallop is a common finding in patients with heart failure with reduced systolic function and is highly specific in the appropriate clinical context. The S3 is a low-frequency early diastolic sound that is best appreciated over the apex of the heart with the bell of the stethoscope (Figure 4-3).4,5

FIGURE 4-3 Phonocardiographic tracing of an S3 gallop recorded over the apex (heart rate 100 beats per minute).

What pharmacologic agents improve symptoms in patients with heart failure with reduced systolic function, regardless of underlying etiology? What pharmacologic agents improve survival in patients with heart failure with reduced systolic function, regardless of underlying etiology? The causes of heart failure with reduced left ventricular systolic function can be separated into which general subcategories?

Patients with heart failure with reduced systolic function from any cause experience improved symptoms with the use of diuretics (when indicated), β-blockers, angiotensin-converting enzyme (ACE) inhibitors or angiotensin II receptor blockers (ARBs), the combination of an ARB and an angiotensin receptor-neprilysin inhibitor (ARNi), the combination of hydralazine and a nitrate, digoxin, and aldosterone antagonists. Decisions regarding choice of agents depend on patient-specific factors (eg, renal function) and disease- specific factors (eg, stage and class of heart failure).1,6 Survival is improved in patients with heart failure with reduced systolic function from any cause with the use of certain β-blocker (eg, metoprolol succinate), ACE inhibitor, ARB, ARNi, the combination of hydralazine and a nitrate (particularly in black patients), and aldosterone antagonist. Decisions regarding choice of agents depend on patient-specific factors (eg, renal 
function) and disease-specific factors (eg, stage and class of heart failure).1,6

The causes of heart failure with reduced left ventricular systolic function can be separated into the following subcategories: cardiovascular, toxic, infectious, and other.

102

103

Cardiovascular Causes of Heart Failure With Reduced Systolic Function

What are the cardiovascular causes of heart failure with reduced systolic function?

The presence of Ischemic cardiomyopathy. electrocardiographic Q waves are a clue to this underlying condition. The “go-fasts.” Tachyarrhythmia-induced cardiomyopathy. A valvular condition Aortic regurgitation. associated with wide pulse pressure. Associated with a Mitral regurgitation. holosystolic murmur over the apex that radiates to the axilla and increases in intensity with handgrip. Two causes of high- Chronic anemia and shunt. output heart failure.

How is ischemic Ischemic cardiomyopathy is defined as left ventricular systolic dysfunction with at least one of the following: (1) a cardiomyopathy history of prior myocardial revascularization or myocardial infarction; (2) >75% stenosis of the left main or left defined? anterior descending coronary arteries; or (3) 2 vessels or more with >75% stenosis.7 What is Tachyarrhythmia-induced cardiomyopathy describes the development of left ventricular dysfunction related to tachyarrhythmia- chronic tachyarrhythmia that improves or resolves after the tachyarrhythmia is controlled (usually within 4 weeks). induced Atrial fibrillation with rapid ventricular response is the most common cause of tachyarrhythmia-induced cardiomyopathy? cardiomyopathy. Other causative tachyarrhythmias include atrial flutter, atrial tachycardia, reentrant supraventricular tachycardia, frequent premature ventricular contractions, and ventricular tachycardia.8,9 What are the The murmur of aortic regurgitation typically begins early in diastole, is decrescendo in shape, and is best heard over characteristics of the third intercostal space along the left sternal border (Erb’s point). Maneuvers that increase blood flow to the heart the murmur of (eg, moving from standing to squatting) can intensify the murmur. There is often an associated systolic ejection aortic regurgitation? murmur that occurs as the regurgitant bolus of blood generates turbulence on its way back through the aortic valve (technically a flow murmur). Aortic regurgitation may be associated with a low-pitched, “blubbering,” mid-to-late diastolic murmur heard over the apex, known as the Austin Flint murmur. Severe aortic regurgitation is associated with numerous peripheral findings (eg, Corrigan’s pulse [a bounding carotid pulse]). Acute regurgitant lesions can be associated with preserved left ventricular systolic function.10 What is the most The most common cause of primary mitral regurgitation in the industrialized world is mitral valve prolapse common cause of secondary to myxomatous degeneration (ie, degenerative changes of the tissues of the mitral valve and chordae primary mitral tendineae, usually idiopathic in nature).11 regurgitation in the industrialized world? What are the Physical findings of high-output states may include elevated JVP, warm extremities, widened pulse pressure (with physical findings of associated findings such as Quincke’s pulse or bounding pulses), hyperdynamic precordium, and a systolic flow high-output states? murmur. Other findings of heart failure may also be present.12

104

Toxic Causes of Heart Failure With Reduced Systolic Function

What are the toxic causes of heart failure with reduced systolic function?

A middle-aged man with a Alcohol. long history of “morning shakes” develops dyspnea with exertion, orthopnea, and elevated JVP. Often snorted. Cocaine. Known on the street as Amphetamines. “speed.” These agents are used for Anthracycline chemotherapeutic agents (eg, doxorubicin). the treatment of malignancies such as breast cancer, leukemia, and lymphoma. An Endocrinopathy. Thyrotoxicosis.

What threshold of alcohol consumption is associated with the development of cardiomyopathy?

How does the acute management of myocardial infarction change when it is related to acute cocaine toxicity? Is methamphetamine-associated cardiomyopathy reversible? How common is anthracycline-associated cardiomyopathy?

What cardiovascular conditions are associated with thyrotoxicosis?

Mild to moderate alcohol consumption is thought to be protective against the development of heart failure (the low point of a J-shaped curve). The risk of alcoholic cardiomyopathy is Increased in those who consume >90 g of alcohol (7-8 drinks) per day for >5 years. It is most common in men 30 to 55 years of age who have consumed heavy amounts of alcohol for >10 years. Only 15% of patients with alcoholic cardiomyopathy are women. Biventricular failure is typical.1 In patients with myocardial ischemia or infarction related to cocaine use, β-blocker medications must be avoided, as unopposed α-receptor stimulation can worsen vasospasm. Cocaine can contribute to the development of heart failure in a number of ways, including infarction related to vasospasm, premature coronary artery disease, vasculitis, and dilated cardiomyopathy. Up to one-fifth of asymptomatic cocaine abusers may have left ventricular dysfunction.1 Methamphetamine-associated cardiomyopathy is reversible if it is recognized early and there is no delay in treatment. The mechanism of heart failure in these patients is thought to be multifactorial with contributions from vasospasm, direct myocyte toxicity, and catecholamine excess. Cor pulmonale related to pulmonary hypertension can also develop in methamphetamine users.13 Overall, an estimated 10% of patients treated with anthracyclines will develop cardiomyopathy within 5 years of completing treatment, with most cases developing within 1 year. All patients undergoing treatment with anthracycline agents should be monitored for the development of cardiotoxicity. There is potential for reversibility if the diagnosis is made early, and treatment is promptly initiated.14 Heart failure typically occurs in hyperthyroid patients with coexistent atrial fibrillation. Other cardiovascular manifestations of hyperthyroidism include pulmonary hypertension and valvular heart disease (usually functional mitral and tricuspid regurgitation). The cardiovascular conditions associated with hyperthyroidism generally reverse with adequate treatment.15

105

Infectious Causes of Heart Failure With Reduced Systolic Function

What are the infectious causes of heart failure with reduced systolic function?

This condition is most often caused by viral infection and typically presents with chest pain, troponin elevation, and diffuse ST-segment elevation. This infectious disease is endemic in South and Central America and is transmitted through the bite of a triatomine bug, also known as the “kissing bug.” This viral infection is highly prevalent in sub-Saharan Africa and is associated with an elevated serum protein gap. Treatment for this systemic condition often involves intravenous fluids, broad-spectrum antibiotics, and vasopressor medications.

What proportion of patients with acute myocarditis will go on to develop chronic heart failure?

What proportion of patients with acute Chagas disease will progress to chronic Chagas disease with associated cardiomyopathy? How has antiretroviral therapy (ART) changed the characteristics of HIV- associated cardiomyopathy?

What is the prognosis of sepsis-associated cardiomyopathy?

Myocarditis.

Chagas disease (caused by Trypanosoma cruzi).16

Human immunodeficiency virus (HIV).17

Sepsis.

Approximately one-third of patients with acute myocarditis develop chronic dilated cardiomyopathy, which is associated with a poor prognosis. In addition to viruses, myocarditis can also be caused by other infectious organisms (eg, Staphylococcus aureus), systemic diseases (eg, systemic lupus erythematosus), and toxins (eg, amphetamines). The clinical presentation can vary considerably and endomyocardial biopsy is the diagnostic gold standard.18 Approximately one-third of patients with acute Chagas disease will progress to the chronic form with associated cardiomyopathy. It is the leading cause of nonischemic cardiomyopathy in Latin America.16

In the pre-ART era, HIV-associated cardiomyopathy was characterized by severe systolic dysfunction and grim prognosis. In the developing world, where ART is not widely available, this type of presentation remains common. In populations where ART is widely used, HIV-associated cardiomyopathy has become less prevalent. When it does occur, it more commonly manifests with diastolic dysfunction. Notably, ART is associated with a higher incidence of coronary artery disease.17 Sepsis-associated cardiomyopathy typically resolves within 7 to 10 days. Although sepsis can be a cause of Takotsubo cardiomyopathy, sepsis-associated cardiomyopathy is a distinct entity. Initial management is the same as in sepsis without cardiomyopathy, with careful attention to volume status.19

106

Other Causes of Heart Failure With Reduced Systolic Function

What are the other causes of heart failure with reduced systolic function?

Always take a family history in a patient Familial dilated cardiomyopathy. presenting with heart failure. This cause of cardiomyopathy only occurs in Peripartum cardiomyopathy. women. “Wet beriberi.” Thiamine deficiency. “Broken-heart” syndrome. Takotsubo cardiomyopathy. Jaundice, spider angiomas, and ascites. Cirrhosis. A 51-year-old woman with a history of carpal Acromegaly. tunnel syndrome presents with heart failure, and her hand feels large, doughy, and moist on handshake. Painful bones and elevated serum alkaline Paget disease. phosphatase. A primary disorder of the muscle. Muscular dystrophy. A middle-aged man develops heart failure Giant cell myocarditis. with reduced systolic function of unknown etiology, and endomyocardial biopsy reveals the presence of multinucleated giant cells. An underlying cause cannot be identified Idiopathic dilated cardiomyopathy. despite a complete workup.

What criteria are used to diagnose familial dilated cardiomyopathy?

When does peripartum cardiomyopathy usually present?

What nutritional deficiencies are associated with heart failure with reduced systolic function? What echocardiographic finding is characteristic of Takotsubo cardiomyopathy? What is the mechanism of heart failure associated with cirrhosis, acromegaly, and Paget disease? What types of muscular dystrophy are associated with heart failure with reduced systolic function? What is the treatment and prognosis for giant cell myocarditis?

How common is idiopathic dilated cardiomyopathy?

Familial dilated cardiomyopathy can be diagnosed in an individual with known idiopathic dilated cardiomyopathy and at least one of the following: (1) at least 1 relative also diagnosed with idiopathic dilated cardiomyopathy, or (2) at least 1 first-degree relative with an unexplained sudden death under 35 years of age.20 The majority of patients with peripartum cardiomyopathy (approximately 80%) present within 3 months of delivery; 10% present during the last month of pregnancy, and 10% present 4 to 5 months postpartum.21 Deficiencies of thiamine, carnitine, selenium, zinc, and copper can result in heart failure with reduced systolic function.22 Echocardiographic apical ballooning in association with basilar hyperkinesis is characteristic of Takotsubo cardiomyopathy. Cirrhosis, acromegaly, and Paget disease cause heart failure as a result of the high-output physiologic state associated with these conditions. Heart failure with reduced systolic function can be associated with Duchenne muscular dystrophy, Becker muscular dystrophy, Emery-Dreifuss muscular dystrophy, limb-girdle muscular dystrophy, and myotonic dystrophy.23 Giant cell myocarditis is treated with combinations of immunosuppressive medications including glucocorticoids, azathioprine, and cyclosporine. Transplant-free survival is estimated to be 70% at 1 year, and 50% at 5 years from symptom onset. A significant proportion of survivors go on to experience sustained ventricular tachyarrhythmias.24 Approximately one-half of all cases of dilated cardiomyopathy remain idiopathic.25

107

108

Heart Failure With Preserved Left Ventricular Systolic Function

What type of myocardial hypertrophy is associated with heart failure with preserved systolic function? Is heart failure with preserved systolic function associated with an enlarged cardiac silhouette on chest radiography? Is heart failure with preserved systolic function typically associated with dilated or restrictive cardiomyopathy? What extra heart sound is commonly associated with heart failure with preserved systolic function?

Heart failure with preserved systolic function is associated with concentric hypertrophy. The chambers of the heart remain similar or decrease in size, but the myocardial walls thicken (see Figure 4-2). Concentric hypertrophy is generally not associated with an enlarged cardiac silhouette on chest radiography.

Heart failure with preserved systolic function is typically associated with restrictive cardiomyopathy. Concentric hypertrophy results in thickened ventricular myocardium with preserved contractility but impaired diastolic filling (ie, compliance is Decreased). An S4 gallop is a common finding in patients with heart failure with preserved systolic function. The S4 is a low-frequency late diastolic sound that is best appreciated over the apex of the heart with the bell of the stethoscope (Figure 4-4).5,10

FIGURE 4-4 Phonocardiographic tracing of an S4 gallop recorded over the apex (heart rate 100 beats per minute).

What is the treatment for heart failure with preserved systolic function?

The causes of heart failure with preserved left ventricular systolic function can be separated into which general subcategories?

Unlike the myriad pharmacologic agents that improve mortality in patients with heart failure with reduced systolic function, no such agents have been proven effective in patients with heart failure with preserved systolic function. Treatment focuses on underlying or associated conditions (eg, hypertension) and symptom management (eg, diuretics).1 The causes of heart failure with preserved left ventricular systolic function can be separated into the following subcategories: Increased afterload, valvular disease, infiltrative disorders, genetic conditions, and other.

109

Causes of Heart Failure With Preserved Systolic Function Related to Increased Afterload

What is afterload in cardiac physiology?

For cardiac muscle, afterload is the force against which the myocardial fibers contract during systole. This force is a product of left ventricular systolic pressure and the internal dimension of the left ventricular cavity.26

What are the causes of heart failure with preserved systolic function related to Increased afterload?

Colloquially referred to as the “silent killer.” Right-sided heart failure related to the lungs or pulmonary vessels. Associated with hypertrophic cardiomyopathy. Brachial-femoral pulse delay and rib notching on chest radiography.

How common is hypertension in patients with heart failure with preserved systolic function? What is the final common pathway of all processes that lead to cor pulmonale? How does the quality of the murmur associated with hypertrophic obstructive cardiomyopathy change with Valsalva maneuver? What are the management strategies for patients with coarctation of the aorta?

Hypertension. Cor pulmonale.

Hypertrophic obstructive cardiomyopathy (HOCM).

Coarctation of the aorta.

Hypertension is present in the vast majority of patients with heart failure with preserved systolic function. The use of β-blockers, ACE inhibitors, and ARBs to control blood pressure in patients with heart failure with preserved systolic function is reasonable. However, no particular class of antihypertensive medications has been shown to improve outcomes in these patients.1,2 Pulmonary hypertension, which increases afterload of the right ventricle, is the final common pathway of all processes that lead to cor pulmonale. The left ventricular outflow tract obstruction of HOCM is dynamic, varying according to several factors, including cardiac preload. When preload is Increased, the degree of obstruction is Decreased; when preload is Decreased, the degree of obstruction is Increased. Preload is Decreased during the straining phase of the Valsalva maneuver, which leads to an increase in the degree of outflow obstruction with an associated increase in the intensity of the murmur.10 In patients with coarctation of the aorta, hypertension should be controlled with β-blockers, ACE inhibitors, or ARBs as first-line medications. Intervention (eg, percutaneous catheter intervention or surgical repair) should be considered in patients with a peak-to-peak coarctation gradient ≥20 mm Hg or in those with a gradient <20 mm Hg who have evidence of significant collateral blood flow (which can decrease the gradient and mask severe obstruction).27

110

Valvular Causes of Heart Failure With Preserved Systolic Function

What general type of valvular lesion is associated with concentric hypertrophy and heart failure with preserved systolic function?

In general, concentric hypertrophy and heart failure with preserved systolic function are associated with stenotic valvular lesions. In contrast, eccentric hypertrophy and heart failure with reduced systolic function are associated with regurgitant valvular lesions. Right-sided valvular lesions, both regurgitant and stenotic, are generally associated with preserved left ventricular function.

What are the valvular causes of heart failure with preserved systolic function?

A late-peaking crescendo-decrescendo systolic murmur best heard over the right upper sternal border with radiation to the carotids and the apex where the murmur takes on a musical quality (ie, Gallavardin phenomenon). A low-pitched rumbling diastolic murmur with presystolic accentuation. A 26-year-old woman with a history of intravenous drug use presents with fever and heart failure and is found to have a holosystolic murmur best heard over the left lower sternal border that augments with inspiration (ie, Carvallo’s sign). Similar in quality to aortic stenosis, but Carvallo’s sign is present. A short, late diastolic murmur with positive Carvallo’s sign. A decrescendo diastolic murmur with positive Carvallo’s sign.

Aortic stenosis.

Mitral stenosis. Tricuspid regurgitation.

Pulmonic stenosis. Tricuspid stenosis. Pulmonic regurgitation.

What are the echocardiographic criteria for severe Severe aortic stenosis is defined by an aortic jet velocity >4.0 m/s or mean gradient aortic stenosis? >40 mm Hg. Aortic valve area is typically <1 cm2 but this finding is not required for diagnosis.28 Why is mitral stenosis often associated with Mitral stenosis is associated with atrial fibrillation in almost half of all cases, and embolic events such as stroke and renal infarction? the risk of embolic events in the setting of valvular atrial fibrillation is higher than in patients with atrial fibrillation alone.29 What characteristic finding of the jugular venous Severe tricuspid regurgitation causes a CV fusion wave in the jugular venous waveform is associated with severe tricuspid waveform, known as Lancisi’s sign. For a video of Lancisi’s sign, see the associated regurgitation? reference.30 What causes pulmonic stenosis? Pulmonic stenosis is almost always congenital in nature and is more common in males. When mild (ie, transpulmonary gradient <25 mm Hg), the clinical course is typically benign. However, more severe degrees of pulmonic stenosis (ie, gradient >50 mm Hg) may necessitate valvuloplasty or valvotomy. Prognosis in patients who undergo such a procedure is excellent.31 What finding of the jugular venous waveform is Tricuspid stenosis causes a giant A wave in the jugular venous waveform. In the associated with tricuspid stenosis? normal waveform, the A wave is caused by an increase in pressure associated with right atrial contraction. When the atrium contracts against a stenotic valve, a large positive pressure wave is generated, producing the giant A wave. For a video of giant A waves, see the associated reference.32 What is a Graham Steell murmur? A Graham Steell murmur describes the murmur of pulmonic regurgitation when it occurs as a result of pulmonary hypertension.10 Aortic stenosis and pulmonic stenosis, which are listed under the valvular category, cause heart failure with preserved systolic function as a result of

111

Increased afterload.

112

Infiltrative Causes of Heart Failure With Preserved Systolic Function

What are the infiltrative causes of heart failure with preserved systolic function?

A middle-aged man with macroglossia and large shoulders. A granulomatous disease. Infiltration of a metallic element. Generalized lymphadenopathy and elevated serum lactate dehydrogenase. Infiltration of “acid-loving” cells.

Amyloidosis.

Sarcoidosis. Iron overload. Lymphoma.

Eosinophilia.

How common is Cardiac involvement can occur in all forms of amyloidosis but is most common in patients with immunoglobulin light cardiac disease in chain (AL) amyloidosis and transthyretin amyloidosis (wild-type and hereditary). Approximately one-half of patients patients with with AL amyloidosis develop cardiac disease, including heart failure in around one-quarter.33,34 amyloidosis? How common is Cardiac sarcoidosis affects up to one-quarter of patients with systemic sarcoidosis. Manifestations most commonly cardiac include conduction disease (eg, Atrioventricular block) and heart failure. Treatment with high-dose glucocorticoids sarcoidosis? may halt progression or reverse heart disease.1 What are the Causes of secondary iron overload include excess dietary intake of iron, severe and chronic hemolysis of any cause, causes of and multiple blood transfusions. secondary iron overload? In addition to Lymphoma can cause the clinical syndrome of heart failure as a result of pericardial infiltration and pericardial myocardial effusion. infiltration, how can lymphoma cause the clinical syndrome of heart failure? What is Löffler Löffler endocarditis (ie, eosinophilic myocarditis) describes the development of endocardial fibrosis in association endocarditis? with hypereosinophilic syndrome (HES), which is defined as persistent hypereosinophilia with an eosinophil count

1500/µL for ≥6 months with evidence of organ damage by eosinophils. HES can be primary (ie, neoplastic), secondary (eg, parasitic infection), or idiopathic (most common). Idiopathic HES is significantly more common in men than women, generally affecting men between the ages of 20 and 50 years. Conditions closely related to Löffler endocarditis include endomyocardial fibrosis and eosinophilic granulomatosis with polyangiitis (EGPA, or Churg-Strauss syndrome). Endomyocardial fibrosis is a disease of the tropics that affects men and women equally; its pathophysiology is unknown.35,36

113

Genetic Causes of Heart Failure With Preserved Systolic Function

What are the genetic causes of heart failure with preserved systolic function?

Cyanosis in the newborn. The leading cause of sudden cardiac death in young athletes; this condition follows an autosomal dominant inheritance pattern. “Bronze diabetes.” Inborn error of metabolism. An X-linked lysosomal storage disorder related to deficiency of the enzyme α-galactosidase A.

Which common congenital heart defect is associated with a fixed split second heart sound (S2) and may lead to right heart failure? How common is heart failure in patients with hypertrophic cardiomyopathy without associated outflow obstruction? What is the treatment of choice in patients with cardiac hemochromatosis?

In patients with glycogen storage disease who present with loss of consciousness, what noncardiac cause should immediately be considered? How common is cardiac involvement in Fabry disease?

Congenital heart disease. Hypertrophic cardiomyopathy.37

Hemochromatosis. Glycogen storage disease. Fabry disease.38

Atrial septal defect (ASD) is associated with a fixed split S2. Over time, as a result of left-to-right shunt, an uncorrected large ASD can lead to right-sided volume overload, flow-related pulmonary hypertension, and eventual right heart failure. Other cardiac manifestations include atrial dysrhythmias such as atrial flutter, atrial fibrillation, and sick sinus syndrome. Small atrial septal defects can remain asymptomatic into the fourth and fifth decades of life.27 Approximately one-third of patients with hypertrophic cardiomyopathy without obstruction develop heart failure with preserved systolic function. Most patients experience a relatively stable course without significant symptoms of heart failure. A small minority of patients develops “burned-out” disease characterized by the conversion to heart failure with reduced systolic function.37

Phlebotomy is first-line treatment for cardiac hemochromatosis in patients without coexistent anemia. Initially, it is typically scheduled every 4 to 14 days as tolerated. Therapeutic targets include ferritin level <50 ng/mL and transferrin saturation <30%. Once the target is reached, maintenance phlebotomy is titrated to keep the ferritin level between 50 and 100 ng/mL and transferrin saturation <50%. Phlebotomy has been shown to improve cardiac parameters in patients with cardiac hemochromatosis, including left ventricular mass and ejection fraction.39 Hypoglycemia is a common manifestation of glycogen storage diseases given the inability to store glycogen, and patients may present with loss of consciousness.

More than half of patients with Fabry disease develop cardiac involvement, most frequently concentric left ventricular hypertrophy. Improvement in cardiac disease can be expected when enzyme replacement therapy is started early.38

114

Other Causes of Heart Failure With Preserved Systolic Function

What are the other causes of heart failure with preserved systolic function?

External cardiac restraint. Pericardial disease. A 42-year-old woman of the Oklahoma Scleroderma (ie, systemic sclerosis).40 Choctaw Native American tribe presents with episodes of discoloration of the fingers on cold exposure and progressive skin tightening. A patient with a history of mediastinal Mediastinal radiation therapy. lymphoma presents with dyspnea and is found to have elevated JVP, Kussmaul’s sign, and marker tattoos on the anterior chest. An underlying cause cannot be identified Idiopathic restrictive cardiomyopathy. despite a complete workup.

What 2 eponymous findings of the jugular venous waveform can be seen in constrictive pericarditis? How does scleroderma affect the heart?

In addition to the myocardium, what other cardiac structures can be affected by radiation therapy?

What is the prognosis of idiopathic restrictive cardiomyopathy?

Constrictive pericarditis is associated with Kussmaul’s sign (a paradoxical increase in JVP with inspiration) and Friedreich’s sign (a sharp and deep Y descent). For a video of Kussmaul’s sign, see the associated reference.41 Cardiac involvement in scleroderma can occur in a variety of ways, including cor pulmonale from lung disease or isolated pulmonary hypertension, acute pericarditis, constrictive pericarditis, pericardial effusion, premature coronary artery disease, myocarditis, nonbacterial thrombotic (marantic) endocarditis, and conduction system abnormalities.42 The spectrum of radiation-induced cardiac disease includes premature coronary artery disease, pericardial disease (eg, pericardial effusion), myocardial disease (eg, myocarditis), valvular disease (eg, aortic stenosis), and conduction system abnormalities (eg, Atrioventricular block).43 Idiopathic restrictive cardiomyopathy is associated with poor prognosis: overall 5-year survival in patients of all ages is 65%; 10-year survival is 35%. Men older than 70 years of age with idiopathic restrictive cardiomyopathy have a particularly high rate of mortality.44

115 Case Summary A 66-year-old woman with a history of hypertension and carpal

tunnel syndrome is referred to cardiology for evaluation of chronic rt heart failure of unknown etiology and is found to have an extra hea sound on examination, an abnormal ECG, evidence of concentric hypertrophy with preserved systolic function on echocardiography,

diffuse biventricular wall thickening with heterogeneous enhancement with apple-green birefringence under polarized light on on contrast-enhanced cardiac MRI, and positive Congo red staining endomyocardial biopsy.

What is the most likely cause of heart failure in this patient? Amyloidosis.

116

Bonus Questions

Which parts of the historyfand physical e Amyloidosis is often associated with carpal tunnel syndrome as a result of soft tissue infiltration. The orthostatic hypotension (with stable diagnosis of amyloidosis? examination in this case o fer clues to th heart rate) in this case is suggestive of autonomic peripheral neuropathy, which is also associated with amyloidosis. 33 What is the mostilikely source of the extra Extra heart sounds that occurinear S1 include split S1, S4 gallop, and ejection clicks. Theiextra sound in this case is most likely an S4 heart sound in th s case? gallop, based on location, pitch, and clinical history (see Figure 4-4). An S4 often occurs n the setting of restrictive cardiomyopathy, which is associated with card ac amyloidosis. 10 Which electrocardiographic feature in t The low-voltage QRS complexes on the ECG in this case (see Figure 4-1) would be highly atypical of hypertensive heart disease, which i case argues against hypertension as thehis usually associated with concentric left ventricular hypertrophy and Increased voltage. Infiltrative disorders, such as amyloidosis, on thes underlying cause of heart failure? other hand, are associated with Decreased QRS voltage on ECG. What are all of the relevant The ECG in this case (see Figure 4-1) demonstrates reduced limb lead QRS voltage (all limb leads ≤0.5 mV) with preserved precordia 33 electrocardiographic findings in this case? voltage, P-wave prominence, and poor R-wave progression in the precordial leads (ie, pseudoinfarction pattern). These findings are l characteristic of cardiac amyloidosis. 22,33 What biochemical tests are helpful when In the appropriate clinical setting, the presence of elevated brain natriuretic peptide (BNP) or its N-terminal fragment (NT-pro-BNP) and cardiac amyloidosis is suspected? cardiac troponins is suggestive of cardiac amyloidosis and is associated with a significantly Increased rate of mortality in these patients. 33 How can cardiac MRI distinguishitheom that On cardiac MRI, late contrast enhancement of the thickened myocardium is highly characteristic of cardiac amyloidosis but is absent in of long-standing hypertension? hypertrophy related to amyloidos s fr patients with myocardial hypertrophy related to hypertension. 33 What is the significance of the case? The histologic demonstration of extracellular deposits with Congo red staining that turn apple-green in color under polarized light is What is the prognosis of cardiac In general, cardiac amyloidosis is associatedlwith a poor prognosis. The median survival of patients with AL.amyloidosis without cardiac endomyocardial biopsy in this pathognomonic for amyloidosis and is the diagnostic gold standard. 33 amyloidosis? involvement is 4 years. Median survival dec ines to 8 months in patients with advanced cardiac involvement 33

117 Key Points

structural or functional impairment of ventricular filling, or the Heart failure is a clinical syndrome that develops as a result of ejection of blood. Symptoms of heart failure include dyspnea, cough, orthopnea,

paroxysmal nocturnal dyspnea, fatigue or lethargy, weight gain, Physical findings of right-sided heart failure include elevated JVP, nausea, early satiety, and abdominal discomfort. right ventricular heave, right-sided gallop, ascites, and lower extremity edema.

Physical findings of left-sided heart failure,include end-inspiratory crepitant rales on auscultation of the lungs left-sided gallop, laterally displaced apical impulse, pulsus alternans, and cool extremities.

The lungs may be clear in the context of chronic left-sided heart systolic function or preserved left ventricularlsystolic function Heart failure with reduced systolic function is associated with systolic function is associated with concentric hypertrophy. separated into the following subcategories: cardiovascular, toxic, valvular disease, infiltrative disorders, genetic conditions, and ,be survival in patients with heart failure with reducedisystolicth failure. Heart failure can be associated with reduced eft ventricular (ejection fraction >50%). eccentric hypertrophy, whereas heart failure with preserved The causes of heart failure with reduced systolic function can be infectious, and other. The causes of heart failure with preserved systolic function can separated into the following subcategories: Increased afterload other. Specific pharmacologic agents have been shown to mprove function; outcomes have been largely neutral in patients wi preserved systolic function.

118

References 1. Yancy CW, Jessup M, Bozkurt B, et al. 2013 ACCF/AHA guideline for the management of heart failure: executive summary: a report of the American College of Cardiology

Foundation/American Heart Association Task Force on practice guidelines. Circulation. 2013;128(16):1810-1852.

  1. Volpe M, McKelvie R, Drexler H. Hypertension as an underlying factor in heart failure with preserved ejection fraction. J Clin Hypertens. 2010;12(4):277-283.

  2. Mensah YB, Mensah K, Asiamah S, et al. Establishing the cardiothoracic ratio using chest radiographs in an indigenous Ghanaian population: a simple tool for

cardiomegaly screening. Ghana Med J. 2015;49(3):159-164. 4. Inamdar AA, Inamdar AC. Heart failure: diagnosis, management and utilization. J Clin Med. 2016;5(7).

  1. Tavel ME. Clinical Phonocardiography and External Pulse Recording. 2nd ed. Chicago, IL: Year Book Medical Publishers, Inc.; 1967.

  2. Yancy CW, Jessup M, Bozkurt B, et al. 2017 ACC/AHA/HFSA focused update of the American College of Cardiology/American Heart Association Task Force on clinical 2013 ACCF/AHA guideline for the management of heart failure: a report of the practice guidelines and the Heart Failure Society of America. Circulation. 2017. Briceno N, Schuster A, Lumley M, Perera D. Ischaemic cardiomyopathy:

  3. pathophysiology, assessment and the role of revascularisation. Heart. 2016;102(5):397- 406.

  4. Anter E, JessupfM, Callans DJ. Atrial fibrillation and heart failure: treatment considerations or a dual epidemic. Circulation. 2009;119(18):2516-2525.

  5. Ellis ER, Josephson ME. What about tachycardia-induced cardiomyopathy? Arrhythm Electrophysiol Rev. 2013;2(2):82-90.

  6. Marriott HJL. Bedside Cardiac Diagnosis. Philadelphia, PA: Lippincott Company; 1993. 11. Enriquez-Sarano M, Akins CW, Vahanian A. Mitral regurgitation. Lancet. 2009;373(9672):1382-1394.

  7. Stern AB, Klemmer PJ. High-output heart failure secondary to arteriovenous fistula. Hemodial Int. 2011;15(1):104-107.

  8. Won S, Hong RA, Shohet RV, Seto TB, Parikh NI. Methamphetamine-associated cardiomyopathy. Clin Cardiol. 2013;36(12):737-742.

  9. Groarke JD, Nohria A. Anthracycline cardiotoxicity: a new paradigm for an old classic. Circulation. 2015;131(22):1946-1949.

  10. Merce J, Ferras S, Oltra C, et al. Cardiovascular abnormalities;in hyperthyroidism: a prospective Doppler echocardiographic study. Am J Med. 2005 118(2):126-131. . Benziger CP, do Carmo GA, Ribeiro AL. Chagas cardiomyopathy: clinical 16presentation and management in the Americas. Cardiol Clin. 2017;35(1):31-47. 17immunodeficiency virus infection: epidemiology, pathophysiology, treatment, and . Remick J, Georgiopoulou V, Marti C, et al. Heart failure in patients with human future research. Circulation. 2014;129(17):1781-1789.

  11. Caforio AL, Pankuweit S, Arbustini E, et al. Current state of knowledge on aetiology, European Society of Cardiology Working Group on Myocardial and Pericardial diagnosis, management, and therapy of myocarditis: a position statement of the Diseases. Eur Heart J. 2013;34(33):2636-2648, 2648a-2648d.

  12. Sato R, Nasu M. A review of sepsis-induced cardiomyopathy. J Intensive Care.

119

2015;3:48. 20. Mestroni L, Maisch B, McKenna WJ, et al. Guidelines for the study of familial dilated cardiomyopathies. Collaborative Research Group of the European Human and Capital

Mobility Project on Familial Dilated Cardiomyopathy. Eur Heart J. 1999;20(2):93-102. 21. Givertz MM. Cardiology patient page: peripartum cardiomyopathy. Circulation. 2013;127(20):e622-e626. . Marinescu V, McCullough PA. Nutritional and micronutrient determinants of

22idiopathic dilated cardiomyopathy: diagnostic and therapeutic implications. Expert Rev Cardiovasc Ther. 2011;9(9):1161-1170.

  1. Verhaert D, Richards K, Rafael-Fortney JA, Raman SV. Cardiac involvement ind patients with muscular dystrophies: magnetic resonance imaging phenotype an genotypic considerations. Circ Cardiovasc Imaging. 2011;4(1):67-76. . Kandolin R, Lehtonen J, Salmenkivi K, Raisanen-Sokolowski A, Lommi J, Kupari M

24Diagnosis, treatment, and outcome of giant-cell myocarditis in the era of combined . immunosuppression. Circ Heart Fail. 2013;6(1):15-22.

  1. Felker GM, Thompson RE, Hare JM, et al. Underlying causes and long-term survival in patients with initially unexplained cardiomyopathy. N Engl J Med.

2000;342(15):1077-1084. 26. Tarazi RC, Levy MN. Cardiac responses to Increased afterload. State-of-the-art review. Hypertension. 1982;4(3 Pt 2):8-18. . Warnes CA, Williams RG, Bashore TM, et al. ACC/AHA 2008 guidelines for the

27management of adults with congenital heart disease: executive summary: a report of practice guidelines (writing committee to develop guidelines for the management of the American College of Cardiology/American Heart Association Task Force on adults with congenital heart disease). Circulation. 2008;118(23):2395-2451. 28. Nishimura RA, Otto CM, Bonow RO, et al. 2014 AHA/ACC guideline for the management of patients with valvular heart disease: a report of the American College

of Cardiology/American Heart Association Task Force on practice guidelines. J Am Coll Cardiol. 2014;63(22):e57-185.

  1. Carabello BA. Modern management of mitral stenosis. Circulation. 2005;112(3):432- 437.

  2. Mansoor AM, Mansoor SE. Images in clinical medicine. Lancisi’s Sign. N Engl J Med. 2016;374(2):e2.

  3. Almeda FQ, Kavinsky CJ, Pophal SG, Klein LW. Pulmonic valvular stenosis in adults: diagnosis and treatment. Catheter Cardiovasc Interv. 2003;60(4):546-557.

  4. Burgess TE, Mansoor AM. Giant a waves. BMJ Case Rep. 2017;2017.

  5. Banypersad SM, Moon JC, Whelan C, Hawkins PN, Wechalekar AD. Updates in cardiac amyloidosis: a review. J Am Heart Assoc. 2012;1(2):e000364.

  6. Hassan W, Al-Sergani H,iMourad W, Tabbaa R. Amyloid heart disease. New frontiers and insights in pathophys ology, diagnosis, and management. Tex Heart Inst J. 2005;32(2):178-184.

  7. Ginsberg F, Parrillo JE. Eosinophilic myocarditis. Heart Fail Clin. 2005;1(3):419-429. 36andlclassification of eosinophilic disorders and related syndromes. J Allergy Clineria . Va ent P, Klion AD, Horny HP, et al. Contemporary consensus proposal on crit Immunol. 2012;130(3):607-612.e9.

  8. Maron BJ, OmmeniSR, SemsarianrC, Spirito P, Olivotto I, Maron MS. Hypertrophic cardiomyopathy: present and futu e, with translation into contemporary cardiovascular med cine. J Am Coll Cardiol. 2014;64(1):83-99.

120

  1. Seydelmann N, WanneriC, Stork S, Ertl G, Weidemann F. Fabry disease and the heart. 39Cardiac involvement in hemochromatosis. Cardiol Rev. 2014;22(2):56-68.an WH. Best Pract Res Clin Endocr nol Metab. 2015;29(2):195-204. . Gulati V, Harikrishnan P, Palaniswamy C, Aronow WS, Jain D, Frishm
  2. Arnett FC, Howard RF, Tan F, et al. Increased prevalence of systemic sclerosis in ape. Native American tribe in Oklahoma. Association with an Amerindian HLA haploty Arthritis Rheum. 1996;39(8):1362-1370.

  3. Mansoor AM, Karlapudi SP. Images in clinical medicine. Kussmaul’s sign. N Engl J Med. 2015;372(2):e3.

  4. Champion HC. The heart in scleroderma. Rheum Dis Clin North Am. 2008;34(1):181- 190; viii.

  5. Yusuf SW, Sami S, Daher IN. Radiation-induced heart disease: a clinical update. Cardiol Res Pract. 2011;2011:317659. . Ammash NM, Seward JB, Bailey KR, Edwards WD, Tajik AJ. Clinical profile and

44outcome of idiopathic restrictive cardiomyopathy. Circulation. 2000;101(21):2490-2496.

121

CHAPTER 5

122

Pericarditis

123 Case: A 32-year-old man with positional chest pain A 32-year-old man presents to the emergency department with shoulders. It is sharp in quality, worsens when the patientilies flat, and painful oral and genital ulcers and inflamed joints over the course of a soCardiaclauscultation is notable for the presence of 3 distinct scratchy increasing episodes of chest discomfort over the course of several days. The pain is located in the center of the chest and rad ates to the improves when he leans forward. He reports a history of recurrent year. unds occurring in concert with the cardiac cycle. Ulcers are present on the ora mucosa, and there are several tender erythematous nodules over the anterior shins. Electrocardiogram (ECG) is shown in Figure 5-1.

FIGURE 5-1

What is the most likely cause of chest pain in this patient?

What is pericarditis? Pericarditis describes inflammation of the pericardium, the fibrous sac that surrounds the heart. Pericarditis is most often an acute process, but acute pericarditis from virtually any cause can become chronic and evolve into constrictive pericarditis. Pericarditis may or may not be associated with a pericardial effusion.1 What are the Symptoms of acute pericarditis may include pleuritic chest pain, nonproductive cough, hiccups, and odynophagia.1 symptoms of acute pericarditis? What are the Pleuritic chest pain is sharp or stabbing in quality, positional, and exacerbated by breathing or coughing. characteristics of pleuritic chest pain? What are the The pleuritic chest pain of acute pericarditis often radiates to the trapezius ridges, is exacerbated by recumbence, characteristic and is improved by leaning forward.1 features of the chest pain associated with acute pericarditis? What are the Physical findings of acute pericarditis may include fever and pericardial friction rub (a high-pitched scratchy sound physical findings of that may have 1, 2, or 3 components). The friction rub occurs as a result of friction between inflamed pericardial acute pericarditis? surfaces and is the cardinal sign of pericarditis.1 What are the Electrocardiographic changes associated with acute pericarditis occur as a result of superficial myocarditis. electrocardiographic Manifestations include diffuse ST-segment elevation, diffuse PR-segment depression, and PR-segment elevation in manifestations of lead aVR. In patients with cardiac tamponade, there may be reduced voltage or the presence of electrical alternans acute pericarditis? (ie, alternating amplitude of the QRS complex).1 Is acute pericarditis Acute pericarditis occurs more commonly in men at a ratio of 4:1.1 more common in men or women? What other Acute pericarditis may be either “dry” or associated with pericardial effusion, the size of which can range from pericardial process trivial without hemodynamic significance, to large and associated with cardiac tamponade.1 can occur in association with acute pericarditis? What is cardiac Cardiac tamponade describes external cardiac compression by abnormal pericardial content (usually fluid) that tamponade? results in hemodynamic changes, the most significant of which is hypotension (Figure 5-2). Virtually any cause of acute pericarditis can be associated with cardiac tamponade.1

124

FIGURE 5-2 In cardiac tamponade, blood or fluid fills the pericardial space, compressing the heart chambers, increasing intracardiac pressure, and obstructing venous return. As blood flow into the ventricles falls, so does cardiac output. (Adapted from Cardiovascular Care Made Incredibly Visual! Philadelphia, PA: Lippincott Williams & Wilkins; 2011.)

Which diagnostic Pericardial fluid may be sampled by performing a pericardiocentesis. The fluid can generally be described as and therapeutic transudative or exudative in nature, depending on fluid characteristics such as protein and lactate dehydrogenase procedure can be (LDH) content. However, unlike fluid from pleural effusions, there is considerable biochemical overlap between performed to pericardial transudates and exudates. Nevertheless, fluid characteristics such as appearance, cholesterol content, sample and drain LDH content, total protein content, cell count, glucose content, Gram stain, culture, and other studies (eg, pericardial fluid? polymerase chain reaction, cytology) can be helpful in establishing the underlying diagnosis.1 What is the utility An enlarged cardiac silhouette on chest radiography, particularly if it is acute, can be a diagnostic clue to the of chest radiography presence of a pericardial effusion. For the cardiac silhouette to be changed, effusions must be large, typically in patients with >250 mL. Significantly smaller effusions can be detected by echocardiography.1 pericardial disease? What is constrictive Constrictive pericarditis describes external cardiac compression by diseased or scarred pericardium (see Figure 5-2). pericarditis? Virtually any cause of acute pericarditis can result in constrictive pericarditis, usually after months to years.1 In patients with Normally, jugular venous pressure decreases with inspiration. In patients with constrictive pericarditis it often constrictive increases with inspiration. This finding is known as Kussmaul’s sign and occurs as a result of Decreased right pericarditis, what ventricular compliance. For a video of Kussmaul’s sign, see the associated reference.2 change in jugular venous pressure can be observed with respiration? The causes of The causes of pericarditis can be separated into the following categories: infectious, malignant, connective tissue pericarditis can be disease (CTD), cardiac, metabolic, and other. separated into which general categories?

125

Infectious Causes of Pericarditis

What are the 3 main groups of organisms that cause infectious pericarditis?

What are the differences in clinical presentation between viral, bacterial, and fungal pericarditis?

Infectious pericarditis can be caused by viral, bacterial, or fungal organisms.

Viral pericarditis is more likely to present with the full spectrum of symptoms and physical findings of acute pericarditis, whereas bacterial (particularly tuberculosis) and fungal causes of acute pericarditis are often insidious and overshadowed by other systemic manifestations of the infection.1

126

Viral Causes of Pericarditis

When does pericarditis usually occur during the course of a viral illness? What is the natural progression of acute viral pericarditis? What are the most common causes of viral pericarditis?

Acute pericarditis may develop during the initial viral infection but more often appears 1 to 3 weeks later (usually following an upper respiratory or Gastrointestinal illness).1 Viral pericarditis is self-limited, typically resolving within 2 weeks.1

Many viruses are capable of causing pericarditis, but the most common offenders include coxsackieviruses, echoviruses, adenoviruses, Epstein-Barr virus (EBV), cytomegalovirus (CMV), human immunodeficiency virus (HIV), influenza A and B viruses, herpes simplex virus, respiratory syncytial virus, and hepatitis A and B viruses.1

Which viruses are most likely to Peak incidence of pericarditis during the spring and fall seasons is characteristic of enteroviruses, cause pericarditis during the spring particularly group A and B coxsackieviruses, adenoviruses, rhinoviruses, echoviruses, and and fall seasons? influenza viruses.1 What characteristics make patients Pericarditis related to EBV and CMV tends to occur in patients with compromised immune particularly susceptible to pericarditis systems.1 related to EBV and CMV? What are the characteristics of Pericardial involvement in patients with HIV/AIDS tends to be a late manifestation. Effusions are pericarditis and/or pericardial small in the vast majority of patients, but tamponade can occur. Survival is significantly better in effusion associated with acquired those without pericardial involvement; only around one-third of patients with pericardial immunodeficiency syndrome involvement related to HIV are alive at 6 months.3 (AIViral infections probably account for the majority of cases of “idiopathic” DS)? pericarditis.1

127

Bacterial Causes of Pericarditis

How does bacterial infection cause pericarditis?

What are the characteristics of pericardial fluid in the setting of acute bacterial pericarditis? How should bacterial pericarditis be managed?

The mechanisms of pericarditis related to bacterial infection include direct invasion of the pericardium from contiguous foci (eg, Infective endocarditis) and hematogenous spread (ie, preexisting noninfectious pericardial effusions can become secondarily infected via hematogenous spread).1 Pericardial fluid related to acute bacterial pericarditis is often turbid in appearance and contains a predominance of polymorphonuclear leukocytes, elevated lactate dehydrogenase, and Decreased glucose.1 Effective management of bacterial pericarditis typically requires the combination of pericardial drainage and systemic antimicrobial agents. Pericardiectomy may be necessary when adhesions and loculations occur.1

What are the bacterial causes of pericarditis?

A common cause of bacterial pericarditis worldwide, particularly in endemic regions. Before the antibiotic era, this organism was a common cause of pericarditis, usually associated with pneumonia. Gram-positive cocci in clusters. Inquiring about travel history and exposure to animals is helpful in identifying the possibility of infection caused by this general group of organisms.

What are the characteristics of tuberculous pericardial effusions? What are the mechanisms of pericarditis caused by streptococcal and staphylococcal species?

What are some of the zoonotic organisms associated with pericarditis?

Mycobacterium tuberculosis (TB).

Streptococcus pneumoniae.

Staphylococcus aureus. Zoonoses.

Tuberculous effusions are typically exudative with elevated protein content and leukocyte count (typically with a lymphocytic predominance >50%). Identification of Mycobacterium by smear, culture, or polymerase chain reaction is sufficient for the diagnosis, but a negative result does not rule out tuberculous pericarditis. High levels of adenosine deaminase activity (ie, >40 U/L) can be suggestive of tuberculous pericarditis.1 A common mechanism of pericarditis related to streptococcal and staphylococcal species is contiguous spread from Infective endocarditis, especially with Streptococcus viridans and Staphylococcus aureus infections. Spread from other intrathoracic foci also occurs, including pneumonia (particularly cases caused by Streptococcus pneumoniae), mediastinitis, wound infection, myocardial abscess (including infected myocardial infarction), and subdiaphragmatic abscess. Hematogenous spread to the pericardium also occurs with bacteremia from streptococcal and staphylococcal species.1 Zoonoses associated with pericarditis include Rickettsia rickettsii (Rocky Mountain spotted fever), Borrelia burgdorferi (Lyme disease), and Coxiella burnetii (Q fever).1

128

Fungal Causes of Pericarditis

What are the 2 epidemiologic categories of fungi?

It is helpful to categorize fungi as either endemic or ubiquitous. Endemic fungi frequently affect both immunocompromised and immunocompetent hosts, whereas ubiquitous fungi predominantly affect immunocompromised hosts.

What are the fungal causes of pericarditis?

This fungus is endemic near the Ohio River Valley and the lower Mississippi River. This fungus is endemic near the San Joaquin River Valley. These 2 ubiquitous fungi are opportunistic and associated with pericarditis in immunocompromised hosts.

Histoplasma capsulatum. Coccidioides immitis. Candida and Aspergillus species.

What is the prognosis of pericarditis caused by In general, pericarditis caused by histoplasmosis is self-limited; however, it may run a histoplasmosis? protracted course in some cases. Most patients recover, although recurrences are common.4 What concurrent site of infection is common Pericardial coccidioidomycosis often occurs with pneumonia.1 in patients with pericardial coccidioidomycosis? What are the predisposing factors for the Risk factors for developing pericardial candidiasis and aspergillosis include recent development of pericardial Candida and antibiotic treatment for bacterial infection, immunocompromised status, and the Aspergillus infections? presence of indwelling catheters.1

129

Malignant Causes of Pericarditis

In addition to routine fluid analysis, what Cytology and flow cytometry of the pericardial fluid, and biopsy of the pericardial tissue additional pericardial studies can be helpful can be helpful in establishing the diagnosis of malignant pericarditis.1 in evaluating for malignancy? In patients presenting with acute pericardial Malignant pericarditis becomes more likely when there is a history of malignancy, cardiac disease, what clinical characteristics tend to tamponade at presentation, a lack of response to nonsteroidal anti-inflammatory drugs, favor underlying malignancy? and recurrent pericarditis.5 What are the 3 ways in which malignancy Malignant pericarditis can occur as a result of metastatic disease (most common), reaction can cause pericarditis? to distant malignancy (ie, non-neoplastic pericardial effusion associated with malignancy elsewhere in the body), and primary pericardial tumor (rare).1

Which malignancies most commonly metastasize to the pericardium? What are the 2 most common types of primary pericardial malignancy?

Malignancies that most commonly metastasize to the pericardium include melanoma, lymphoma, leukemia, and lung, breast, and esophageal cancer.6 Mesotheliomas and sarcomas are the most common primary pericardial malignancies. These tumors tend to be aggressive, often spreading through the pericardium to invade the myocardium.1

130

Pericarditis Related to Connective Tissue Disease

Which gender is disproportionally affected by pericardial involvement of connective tissue disease?

Although connective tissue diseases tend to be more prevalent in women, pericardial involvement related to these conditions occurs more frequently in men.1

What are the connective tissue diseases that cause pericarditis?

A 48-year-old woman with symmetric inflammatory polyarticular arthritis is found to have serum anti-cyclic citrullinated peptide (anti-CCP) antibodies. A 31-year-old woman with recurrent episodes of acute pericarditis associated with serum anti–double-stranded DNA antibodies and low complement levels. This disease can be either diffuse or limited, both types of which can be associated with pericardial disease. An overlap syndrome with features of SLE, scleroderma, and dermatomyositis/ polymyositis. Oligoarticular inflammatory arthritis, often involving the axial skeleton, with negative serum rheumatoid factor. May be associated with palpable purpura.

What are the clinical features of the pericardial involvement that occurs with rheumatoid arthritis?

What are the clinical features of the peri- cardial involvement that occurs with systemic lupus erythematosus? What are the clinical features of the pericardial involvement that occurs with scleroderma?

What are the clinical features of the pericardial involvement that occurs with mixed connective tissue disease? Of the seronegative spondyloarthritides, which is most frequently associated with pericarditis?

Which vasculitides are associated with pericarditis?

Rheumatoid arthritis.

Systemic lupus erythematosus (SLE).

Scleroderma (ie, systemic sclerosis).

Mixed connective tissue disease (MCTD).

Seronegative spondyloarthritides.

Vasculitis.

Around half of patients with RA have Increased pericardial fluid on echocardiography, and nearly half have significant pericardial adhesions on autopsy. Patients most commonly present with either an asymptomatic pericardial friction rub or an asymptomatic effusion on echocardiography. Most effusions are serous with low glucose, Increased protein, Increased cholesterol, and Decreased complement.1 Some form of pericarditis develops in the majority of patients with SLE, particularly men. SLE can cause a spectrum of pericardial abnormalities, from large pericardial effusions to constrictive pericarditis. Pericardial involvement is often the first manifestation of SLE and should trigger an investigation to evaluate for the disease in select patients (eg, young women).1 Pericardial involvement is frequent in patients with scleroderma and can take many forms, including acute pericarditis, large pericardial effusion, and constrictive pericarditis. Pericardial effusion is present on echocardiography in close to one-half of patients, and pericardial disease is present in most patients at autopsy. Despite these high rates, most patients do not experience significant clinical manifestations.1 Pericarditis is the most frequent cardiac manifestation of MCTD and can be a presenting feature. The electrocardiographic manifestations of pericarditis (eg, diffuse ST-segment elevation) are more frequent in patients with MCTD than other connective tissue diseases. Prognosis is generally good with most cases being responsive to short courses of glucocorticoid therapy.1 Pericardial involvement occurs frequently in reactive arthritis, particularly acute pericarditis with or without pericardial effusion.1

Among the vasculitides, pericardial involvement is most common in patients with granulomatosis with polyangiitis (GPA, or Wegener’s granulomatosis), but it also occurs with giant cell arteritis, eosinophilic granulomatosis with polyangiitis (EGPA, or Churg-Strauss syndrome), polyarteritis nodosa, and Behçet’s disease.1

131

132

Cardiac Causes of Pericarditis

What are the cardiac causes of pericarditis?

An umbrella term describing the development of Postcardiac injury syndrome (PCIS). pericarditis following various types of cardiac injury. A 54-year-old man with arachnodactyly (Figure 5-3) Aortic dissection. and a high-arched palate presents with tearing substernal chest pain that radiates to the back, and is found to have a blood pressure of 183/98 mm Hg in the right upper extremity and 104/65 mm Hg in the left upper extremity.

FIGURE 5-3 Long and slender fingers (arachnodactyly) in a patient with Marfan syndrome.

What are the various causes of postcardiac injury syndrome?

What are the clinical features of infarct pericarditis?

What are the clinical features of Dressler’s syndrome?

What is the mechanism of pericardial disease associated with aortic dissection?

PCIS can be caused by myocardial infarction (ie, infarct pericarditis), Dressler’s syndrome (ie, postmyocardial infarction syndrome), trauma, and postpericardiotomy syndrome.1 Infarct pericarditis (ie, pericarditis epistenocardica) occurs when there is transmural or near transmural infarction. It is limited to the pericardium adjacent to the zone of infarction and occurs early in the course of myocardial infarction (unlike Dressler’s syndrome, which is delayed). Pericardial friction rubs are usually present and tend to be monophasic, with a peak incidence between the first and third days.1 Dressler’s syndrome is characterized by severe pleuritic chest pain, fever, pericardial friction rub, and elevated erythrocyte sedimentation rate. It can develop even without transmural infarction. Onset is typically 1 week to several months after infarction. Pericardial effusion occurs in around half of patients, and concurrent pleural involvement is common.1 Dissecting aortic aneurysms can rupture into the pericardium, which may lead to sudden death via cardiac tamponade. Pericardial effusion may also develop slowly over a longer period of time (weeks to months), allowing massive amounts of blood (as much as 1500 mL) to encase the heart. Surgical drainage is required.1

133

Metabolic Causes of Pericarditis

What are the metabolic causes of pericarditis?

Asterixis and a pericardial friction rub. Associated with treatment for uremia. The development of pericarditis and associated pericardial effusion in this condition is often slow, mirroring its effect on metabolism.

What are the clinical features of uremic pericarditis?

What is dialysis-related pericarditis?

What are the clinical features of the pericardial involvement that occurs with hypothyroidism?

Uremia. Dialysis-related pericarditis. Hypothyroidism.1

Uremic pericarditis generally does not occur unless blood urea nitrogen levels are >60 mg/dL (although this relationship is not strict). It does not discriminate between the underlying causes of renal failure. The typical electrocardiographic features of pericarditis are often absent. There is an Increased risk of bleeding into the pericardium with associated cardiac tamponade in patients with uremic pericarditis.1 Dialysis-related pericarditis describes the development of pericarditis in dialysis patients despite good biochemical control of renal failure. Its pathogenesis is not known, but it is significantly less common in patients who receive peritoneal dialysis compared with those who receive hemodialysis. Precipitants include inadequate dialysis, volume overload, and systemic infection.1 Pericardial involvement occurs in severe cases of hypothyroidism (ie, myxedema). It typically manifests as a pericardial effusion; signs of pericardial inflammation are almost always absent. Pericardial involvement is virtually always completely reversed with adequate thyroid hormone replacement therapy.1

134

Other Causes of Pericarditis

What are the other causes of pericarditis?

Iatrogenic Medication and radiation therapy. complications. No underlying Idiopathic. cause is identified despite a thorough workup.

What medications are associated with pericarditis?

What are the clinical features of pericardial disease related to radiation therapy? What proportion of cases of acute pericarditis is idiopathic?

Numerous medications can be associated with pericardial disease, typically manifesting as acute pericarditis or inflammatory pericardial effusion. Some of the more widely used agents include penicillins (eg, ampicillin), sulfa drugs, thiazides, amiodarone, procainamide, cyclosporine, sirolimus, minoxidil, hydralazine, and doxorubicin. Anticoagulants and thrombolytics can precipitate bleeding into the pericardial space when there is preexisting pericarditis.1 Radiation therapy for diseases arising in the vicinity of the pericardium, such as mediastinal lymphoma, breast cancer, and lung cancer, frequently leads to pericardial disease. Severity depends on radiation dose, duration of treatment, and extent of the radiation field. While acute pericarditis can develop at the time of therapy, pericardial disease related to radiation is most commonly delayed, sometimes for many years, and most often presents as a chronic effusion or constrictive pericarditis.1 A definitive underlying diagnosis is elusive in around 80% of cases of acute pericarditis. The majority of these cases are likely viral in nature.7,8

135 Case Summary A 32-year-old man with a history of recurrent painful oral and

genital ulcers and arthritis presents with acute-onset pleuritic chest pain and is found to have oral ulcers and tender erythematous

nodules over the anterior shins. What is the most likely cause of chest pain in this patient? Acute pericarditis.

136

Bonus Questions

What is the nature of the The patient in this case has a pericardial friction rub, the cardinal sign of pericarditis. The “scratchy” sounds occur as a result of friction between inflamed extra heart sounds pericardial surfaces and are generally best appreciated with the diaphragm of the stethoscope along the left mid- to lower-sternal border. Rubs may be described in this case? transient and often change with position or respiration. Except when palpable in uremic pericarditis, friction rubs can only be appreciated by auscultation, one of many reasons the stethoscope is an irreplaceable tool in the arsenal of the skilled clinician.1 What cardiac events The complete 3-component friction rub is the result of 2 diastolic events (passive ventricular filling and atrial contraction) and 1 systolic event (ventricular components of the generate the 3 contraction). 1 pericardial friction rub? Whatocardiographicn The ECG in this case (seeiFigure 5-1) demonstrates diffuse ST-segment elevation, diffuse PR-segment depression, and PR-segment elevation in aVR. These What is the most likelyte The patient in this case most likely has Behçet’s disease, given the recurrent oral and genital ulcers, arthralgias/arthritis, and erythema nodosum (see Figure What are clinical ements The pericardium is thetmost common site of cardiac involvement in Behçet’s disease. Manifestations include acute pericarditis, pericardial effusions rangingjor How should this patient Pericarditis related to Behçet’s disease is generally self-limited and responsive to anti-inflammatory medications used to treat the disease itself.1 findings are present i this case? electr findings are consistent w th acute pericarditis. pericarditis in this case? underlying cause of acu 15-3). that occurs with Behçet’ disease? features of the from small and asymp omatic to large with associated tamponade, and constrictive pericarditis. Of note, Behçet’s disease can provoke thromboses of the ma pericardial involv veins, mimicking pericardial constriction. 1 be treated?

137 Key Points

Pericarditis describes inflammation of the pericardium, the fibrous Pericarditis is most often an acute process but can become chronic Acute pericarditis may be “dry” or associated with pericardial hemodynamic significance, to large and associated with cardiac Symptoms of acute pericarditis include pleuritic chest pain, sac that surrounds the heart. and evolve into constrictive pericarditis. effusion, the size of which can range from trivial without tamponade. nonproductive cough, hiccups, and odynophagia. Physical findings of acute pericarditis include fever and

pericardial friction rub, a high-pitched scratchy sound that may have 1, 2, or 3 components. The electrocardiographic findings of acute pericarditis include

diffuse ST-segment elevation, diffuse PR-segment depression, and categories: infectious, malignant, connective tissue disease, g Infectious pericarditis is most often viral in nature, but bacterial PR-segment elevation in lead aVR. The causes of pericarditis can be separated into the followin cardiac, metabolic, and other. and fungal cases do occur. Cytology and flow cytometry of the pericardial fluid, and biopsy

of the pericardial tissue can be helpful in establishing the diagnosis Men are more likely than women to develop pericardial Postcardiac injury syndrome islan umbrella term that describes the The majority of cases of acute pericarditis are idiopathic, which are of malignant pericarditis. involvement from connective tissue disease. development of pericarditis fol owing cardiac injury. Uremia is the most common metabolic cause of pericarditis. most likely undiagnosed viral infections.

138

References 1. Spodick DH. The Pericardium: A Comprehensive Textbook. New York, NY: Marcel Dekker, Inc.; 1997.

  1. Mansoor AM,(Karlapudi SP. Images in clinical medicine. Kussmaul’s sign. N Engl J Med. 2015;372 2):e3.
  2. Heidenreich PA, Eisenberg MJ, Kee LL, et-al. Pericardial effusion in AIDS. Incidence and survival. Circulation. 1995;92(11):3229 3234.
  3. Picardi JL, Kauffman CA, Schwarz J, Holmes JC, Phair JP, Fowler NO. Pericarditis caused by Histoplasma capsulatum. Am J Cardiol. 1976;37(1):82-88.

  4. Imazio M, Demichelis B,lParrini I, et al. Relation of acute pericardial disease to 6. Klatt EC, Heitz DR. Cardiac metastases. Cancer. 1990;65(6):1456-1459. malignancy. Am J Cardio . 2005;95(11):1393-1394.

  5. disease: a prospective series of 231 consecutive patients. Am J Cardiol. 1985;56(10):623- Permanyer-Miralda G, Sagrista-Sauleda J, Soler-Soler J. Primary acute pericardial 630.

  6. Zayas R, Anguita M, Torres F, et al. Incidence of specific etiology and role of methods 1995;75(5):378-382. for specific etiologic diagnosis of primary acute pericarditis. Am J Cardiol.

139

CHAPTER 6

140

Tachycardia

141 Case: A 65-year-old woman with palpitations A 65-year-old woman with a history of coronary artery disease

presents to the emergency department with chest palpitations. She underwent percutaneous coronary intervention with deployment of a left ventricular systolic dysfunction at the time of discharge or at any medications including aspirin, atorvastatin, metoprolol succinate, and had an ST-elevation myocardial infarction at 62 years of age, and drug-eluting stent to the circumflex artery. There was no evidence of time during her follow-up with cardiology. She has been adherent to lisinopril. She began feeling chest palpitations on the day of presentation.iShe has not experienced chest pain or light-headedness. s with intermittent large outward pulsations. No murmurs are 2 Heart rate s regular and 144 beats per minute, and blood pressure i 118/69 mm Hg. Jugular venous pressure is estimated to be 7 cm H O appreciated. The lungs are clear. Electrocardiogram (ECG) is shown in Figure 6-1.

FIGURE 6-1

Serum biomarkers are negative, and repeat transthoracic echocardiography shows an area of lateral wall akinesis but preserved

left ventricular systolic function. Coronary angiography demonstrates What rhythm disturbance is present in this patient? a patent stent in the circumflex and patent native vessels.

What is the path of electrical conduction in the normal heart?

How is heart rate regulated?

What is the definition of tachycardia in adults? What are the 3 basic mechanisms of tachycardia? What is the relationship

In the normal heart, an impulse spontaneously originates from the sinoatrial (SA) node, which is located in the subepicardial surface at the junction of the right atrium and superior vena cava. The impulse propagates through the myocytes of the right and left atria simultaneously before reaching the Atrioventricular (AV) node, which is located in the inferior portion of the right atrium. From there, the impulse is conducted to the bundle of His within the membranous septum, which then separates into the right and left bundle branches supplying the right and left ventricles, respectively (see Figure 1-2). The sympathetic and parasympathetic nervous systems innervate the conduction system of the heart. Parasympathetic tone decreases SA node automaticity and AV node conduction, whereas sympathetic input increases SA node automaticity and AV node conduction.1 The average resting heart rate in adults is 70 beats per minute. Tachycardia is classically defined by a heart rate greater than 100 beats per minute.2,3

Tachycardia can occur as a result of Increased pacemaker automaticity (eg, sinus tachycardia), triggered activity outside of the normal conduction system (eg, ectopic impulses), or reentry (eg, AV nodal reentrant tachycardia [AVNRT]).3 Cardiac output (CO) is equal to the forward stroke volume (SV) of the left ventricle per beat multiplied by heart rate (HR).1CO =SV ×HR

142

between cardia

output and heartc rate? What are the symptoms of tachycardia? What are the physical findings of tachycardia? What are the 2 electrocardiographic categories of tachycardia?

What is the definition of a wide QRS complex? The small boxes on the electrocardiogram represent how many milliseconds?

Patients with tachycardia may be asymptomatic. Symptoms may include palpitations, light-headedness, syncope, chest pain, and dyspnea. The cardinal physical finding of tachycardia is a fast pulse rate, which can be regular or irregular. Additional findings may include hypotension and cool extremities. Tachycardia can be associated with a narrow QRS complex or a wide QRS complex.

A wide QRS complex is defined electrocardiographically as QRS duration >120 ms (see Figure 1-2).

At the standard paper speed of 25 mm/s, each small box (1 mm in width) on the ECG corresponds to 40 ms. Each large box, which is composed of 5 small boxes, represents 200 ms (see Figure 1-2).

143

Narrow-Complex Tachycardia

What are the 2 subcategories of narrow- complex tachycardia?

What are the electrocardiographic characteristics of a regular rhythm?

Narrow-complex tachycardia can be associated with a regular rhythm or an irregular rhythm.

Regular rhythm is defined electrocardiographically by the presence of QRS complexes that are separated by a constant interval (ie, the R-R interval is constant).

144

Narrow-Complex Tachycardia with Regular Rhythm

What are the causes of narrow-complex tachycardia with regular rhythm?

A 34-year-old Sinus tachycardia related to infection. man presents with purulent cough, fever, leukocytosis, and a heart rate of 125 beats per minute. This rhythm Atrial flutter. most commonly originates in the right atrium. Dual AV nodal Atrioventricular nodal reentrant tachycardia (AVNRT). pathways physiology is required for this type of tachycardic rhythm. Wolff- Atrioventricular reentrant tachycardia (AVRT). Parkinson-White syndrome. This rhythm Atrial tachycardia. originates from a focus within the atria rather than the SA node. No visible or Junctional tachycardia. discernible P waves associated with the QRS complexes.

What are the characteristics of sinus tachycardia? What are the characteristics of atrial flutter?

What are the characteristics of Atrioventricular nodal reentrant tachycardia?

Sinus tachycardia is characterized by gradual onset with heart rates generally between 100 and 140 beats per minute (maximum HR is approximately 220 beats per minute minus the patient’s age). Rhythms that are generated from the SA node are electrocardiographically characterized by 
P waves that are morphologically identical and upright (ie, positive) in leads 
I and aVF. Atrial flutter is a type of a reentrant circuit that involves an area near the tricuspid valve in the right atrium, called the cavotricuspid isthmus, as an essential part of its circuit. The atrial rate is typically 240 to 350 beats per minute. Commonly, the atrial rate is 300 beats per minute, and there is 2:1 conduction within the AV node, resulting in a ventricular rate of 150 beats per minute, which can be a clue to the diagnosis.4 AVNRT typically occurs in patients without evidence of structural heart disease. Onset is abrupt with ventricular rates generally between 150 and 250 beats per minute. AVNRT requires dual AV nodal physiology (2 pathways with different electrophysiologic properties), 1 slow (with a shorter refractory period) and 1 fast (with a longer refractory period). Dual pathways are present in up to one-third of the general population. Normal sinus rhythm usually conducts through the fast pathway, whereas competing anterograde/retrograde conduction nullifies transmission through the slow pathway. Typical AVNRT (common) is triggered by atrial premature depolarization with anterograde conduction through the slow pathway (while the fast pathway remains refractory). Atypical AVNRT (uncommon) is triggered by ventricular premature depolarization with retrograde conduction through the slow pathway 
(Figure 6-2).3,5

145

FIGURE 6-2 Model of dual AV nodal pathways physiology in sinus rhythm (left), with an atrial premature beat (APB), which

What are the characteristics of Atrioventricular reentrant tachycardia?

initiates typical “slow-fast” AVNRT (middle), and with a ventricular premature beat (VPB), which initiates atypical “fast-slow” AVNRT (right). (Adapted from Mani BC, Pavri BB. Dual Atrioventricular nodal pathways physiology: a review of relevant anatomy, electrophysiology, and electrocardiographic manifestations. Indian Pacing Electrophysiol J. 
2014;14(1):12-25 .)

AVRT is a type of reentrant tachycardia that requires the presence of a bypass tract (ie, accessory pathway) between the atria and ventricles that is capable of conducting in the anterograde direction, in the retrograde direction, or in both directions. When the reentrant loop is characterized by anterograde conduction down the AV node and retrograde conduction through the bypass tract (orthodromic), the QRS complex is narrow. When it is characterized by anterograde conduction through the bypass tract and retrograde conduction through the AV node (antidromic), the QRS complex is wide. Onset is abrupt with ventricular rates generally between 150 and 250 beats per minute. In sinus rhythm, when there is anterograde conduction down the accessory pathway, an initial slurring of the QRS complex can be seen and is known as a delta wave (Figure 6-3).4

FIGURE 6-3 Sinus rhythm with short PR interval and delta wave (ie, pre-excitation pattern) consistent with the presence of an accessory pathway. The delta wave is positive in all leads except aVR and V1 where it is negative. (From Woods SL, Froelicher ES, Motzer SA, Bridges EJ. Cardiac Nursing. 6th ed. Philadelphia, PA: Wolters Kluwer Health; 2010.)

What are the characteristics of atrial tachycardia?

Junctional tachycardia can occur as a result of what commonly prescribed cardiac medication?

Atrial tachycardia is defined as an atrial rhythm with a rate greater than 100 beats per minute originating outside the SA node. Mechanisms can be reentry (micro- or macroreentrant circuits) or focal activity (automatic or triggered) within the atria. Onset is abrupt with ventricular rates generally between 150 and 250 beats per minute. Atrial tachycardia tends to occur in repetitive short bursts, usually preceded by a “warm-up” period in which the atrial rate increases over a period of 5 to 10 seconds before stabilizing.4 Junctional tachycardia is associated with digitalis toxicity.6

146

Narrow-Complex Tachycardia with Irregular Rhythm

What are the causes of narrow-complex tachycardia with Irregular Rhythm?

No P waves on ECG. This rhythm is strongly associated with lung disease, particularly chronic obstructive pulmonary disease. You are confused because you identify the presence of flutter waves on ECG, but the rhythm is irregular.

Atrial fibrillation. Multifocal atrial tachycardia (MAT).7

Atrial flutter with variable AV conduction.

What are the Atrial fibrillation is the most common dysrhythmia. It is the result of multiple electrical wavelets in the atria occurring characteristics of simultaneously so that there is no coordinated atrial contraction. Risk factors include older age, male sex, hypertension, atrial and underlying cardiac disease. The onset of rapid heart rates can be sudden, particularly in patients with acute atrial fibrillation? fibrillation, or gradual, which usually occurs in patients with chronic atrial fibrillation, with ventricular rates generally between 100 and 220 beats per minute. In older patients with chronic atrial fibrillation, rate control and rhythm control strategies are associated with equivalent outcomes.4,8 What are the MAT is the result of Increased atrial automaticity, most commonly related to hypoxia, Increased atrial pressure, or characteristics of theophylline treatment. Onset is gradual with ventricular rates generally between 100 and 150 beats per minute. MAT multifocal atrial is defined electrocardiographically by the following features: atrial rate greater than 100 beats per minute; at least 3 tachycardia? morphologically distinct P waves associated with variable P-P intervals; and an isoelectric baseline between P waves.4,9 Does flutter In atrial flutter with variable AV conduction, flutter wave morphology remains the same, but the rate at which the wave waves conduct through the AV node changes. Flutter waves can be identified between the QRS complexes to morphology determine the flutter rate (the interval between 2 flutter waves). The ratio of AV conduction can then be calculated by change in dividing the flutter rate by the ventricular rate. patients with atrial flutter with variable AV conduction?

147

Wide-Complex Tachycardia

What are the 2 types of QRS morphologies associated with wide-complex tachycardia?

Wide-complex tachycardia can be associated with uniform QRS morphology (monomorphic) or variable QRS morphology (polymorphic).

148

Monomorphic Wide-Complex Tachycardia

What are the 2 subcategories of monomorphic wide- complex tachycardia?

Monomorphic wide-complex tachycardia can be associated with a regular rhythm or an irregular rhythm.

149

Monomorphic Wide-Complex Tachycardia with Regular Rhythm

What are the causes of monomorphic wide-complex tachycardia with regular rhythm?

Often associated with a myocardial scar. These rhythms originate above the ventricles. This rhythm is generated from a device.

Monomorphic ventricular tachycardia (VT).

Regular supraventricular tachycardia (SVT) with a baseline wide QRS complex (ie, bundle branch block) and regular SVT with aberrant conduction (eg, rate related). Pacemaker-facilitated tachycardia.

What are the Monomorphic VT most commonly occurs in association with myocardial scarring (from prior myocardial characteristics of infarction), but also occurs in the setting of dilated cardiomyopathy, prior cardiac surgery, infiltrative disorders, monomorphic VT? and hypertrophic cardiomyopathy. It may also occur in the structurally normal heart. It is sustained when it lasts at least 30 seconds or is associated with hemodynamic instability. Patients with hemodynamic instability should immediately be treated with synchronized direct current cardioversion.3,10,11 What is the SVT with a baseline wide QRS complex refers to a baseline block in one of the bundle branches (right bundle branch difference between or left bundle branch). SVT with aberrancy, on the other hand, refers to a “functional” block that occurs in one of SVT with a baseline the bundle branches only in certain circumstances (eg, tachycardia-related aberrancy).12 wide QRS complex and SVT with aberrancy? What validated The Brugada criteria is the most commonly used ECG algorithm to distinguish VT from either regular SVT with a electrocardiographic baseline wide QRS complex or regular SVT with aberrancy (Figure 6-4). The majority of wide-complex tachycardias algorithm can 13 are VTs. distinguish VT from either regular SVT with a baseline wide QRS complex or regular SVT with aberrancy?

FIGURE 6-4 Brugada criteria for distinguishing ventricular tachycardia (VT) from either supraventricular tachycardia

What are the mechanisms of pacemaker- facilitated

(SVT) with a baseline wide QRS complex or SVT with aberrancy. (Atachycardia with a wide QRS complex. Circulation. 1991;83(5):1649-1659. Copyright © 1991, American Heart Association.)lar dapted from Brugada P, Brugada J, Mont L, Smeets J, Andries EW. A new approach to the differential diagnosis of a regu

Pacemaker-facilitated tachycardia occurs either as a result of pacemaker-mediated tachycardia (PMT), or as a result of tracking of an atrial rhythm. Between the two, PMT is more common. It occurs in patients with dual-chamber pacemakers and intact retrograde conduction when a ventricular contraction (either spontaneous or paced) is conducted retrograde through the AV node, where it depolarizes the atria. The retrograde P wave is then sensed by

150

tachycardia? the atrial lead. The pacemaker waits for the programmed AV interval and then triggers ventricular pacing. However, retrograde conduction through the AV node occurs again, followed by paced ventricular activation in an endless loop. Pacemaker-facilitated tachycardia due to tracking of an atrial rhythm occurs when a supraventricular tachyarrhythmia (eg,iatrial tachycardia) is sensed by the atrial lead, which attempts to maintain Atrioventricular synchrony by trigger ng ventricular pacing at the same atrial rate. 14

151

Monomorphic Wide-Complex Tachycardia with Irregular Rhythm

What are the causes of monomorphic wide-complex tachycardia with Irregular Rhythm?

These rhythms originate above the ventricles.

What are some examples of irregular SVT with a wide QRS complex? Does aberrant conduction occur more commonly in the pattern of a right or left bundle branch block?

Irregular SVT with a baseline wide QRS complex and irregular SVT with aberrant conduction.

Examples of irregular SVT with a wide QRS complex include the following: atrial fibrillation with an accessory pathway (known as preexcited atrial fibrillation); atrial flutter with variable conduction and a baseline bundle branch block; and multifocal atrial tachycardia with a baseline bundle branch block. Aberrant conduction most commonly occurs in a right bundle branch block pattern because the refractory period of the right bundle is longer than that of the left.15

152

Polymorphic Wide-Complex Tachycardia

What are the causes of polymorphic wide-complex tachycardia?

A ventricular rhythm with variable QRS morphology most commonly associated with ischemia. A type of polymorphic VT described by the phrase, “twisting around the points.”

Polymorphic VT.

Torsades de pointes (Figure 6-5).3

FIGURE 6-5 Torsades de pointes. The QRS changes from negative to positive polarity and appears to twist around the

Disorganized ventricular electrical activity that is invariably fatal without prompt treatment.

What are the characteristics of polymorphic VT?

What are the characteristics of torsades de pointes?

What are the characteristics of ventricular fibrillation?

isoelectric line. (From Huff J. ECG Workout Exercises in Arrythmia Interpretation. 7th ed. Philadelphia, PA: Wolters Kluwer; 2017.)

Ventricular fibrillation (VF). VF does not produce true QRS complexes per se because there is no coordinated ventricular contraction.

Polymorphic VT most commonly occurs in the context of acute ischemia, but also occurs in patients with electrolyte disturbances, long QT syndrome, Brugada syndrome, and structurally normal hearts. Evaluation for underlying coronary artery disease is important in patients with polymorphic VT. Patients with hemodynamic instability should immediately be treated with defibrillation.10,11 Torsades de pointes is a type of polymorphic VT that occurs in the setting of a prolonged QT interval, such as in genetic syndromes or exposure to QT-prolonging medications. It can be short and self-terminating, causing palpitations and syncope, or if sustained it can deteriorate into VF and cardiac arrest. Intravenous magnesium can be used to terminate torsades de pointes. Patients with hemodynamic instability should immediately be treated with defibrillation.10,16 VF is an unstable and pulseless condition characterized by rapid and disorganized ventricular electrical activity resulting in the absence of coordinated ventricular contraction. The electrocardiographic features of VF include irregular QRS complexes of variable morphology and amplitude. Patients should immediately be treated with defibrillation.10

153 Case Summary infarction was admitted with palpitations and hemodynamically l A 65-year-old woman with a history of ST-elevation myocardia stable tachycardia.

What rhythm disturbance is present in this patient? Monomorphic ventricular tachycardia.

154

Bonus Questions

What are the electrocardiographicin The ECG in this case (see Figure 6-1) demonstrates a wide-complex tachycardia with uniform QRS morphology (monomorphic) and regular What are the possible causes of The differential diagnosis for monomorphic wide-complex tachycardia with regular rhythm includes monomorphic VT, regular SVT with a tachycardia with regular rhythm? narrowed to monomorphic VT or SVT with a wide QRS complex. The Brugada criteria can be used to differentiate these conditions (see Figure After which step in the Brugada criteria The diagnosis of VT is made after the first step in the Brugada algorithm (see Figure 6-4). There is no RS complex in any of the precordial leads. this case? characteristics of the tachycardia rhythm. monomorphic wide-complex baseline wide QRS complex, regular SVT with aberrancy, and PMT. This patient does not have a pacemaker, so the differential diagnosis can be 6-4). is the diagnosis of VT made in this case? What is the most likely cause of? The most common substrate for monomorphic VT is myocardial scarring related to infarction. Indeed, it the most likely explanation in this case What effective strategy can be used to Implantable cardioverter-defibrillators reduce mortality as a method of primary prevention in select patients with reduced systolic function.11 monomorphic VT in this patient given the history of myocardial infarction. 11 prevent sudden death related to VT in patients with heart disease? sustained monomorphic VT withcute of procainamide, and amiodarone. Iflpharmacologic conversion of VT is unsuccessful, synchronized electrical cardioversion can be attempted.11 When is it appropriate to treat a When there is hemodynamic stabi ity, monomorphic VT can be treated with pharmacologic cardioversion, using agents such as lidocaine, pharmacologic cardioversion instead electrical cardioversion? What long-term treatment options are There are a variety of modalit es available to treat patients who survive an episode of sustained monomorphic VT. If the episode was unrel available for patients with a history of to a clear reversible cause (eg,ielectrolyte disturbance), then patients should receivetan implantable cardioverter-defibrillator for secondaryated sustained monomorphic VT? prevention. Patients with recurrent episodes of sustained monomorphic VT can be reated with pharmacologic agents (eg, β-blocker, amiodarone) and radiofrequency catheter ablation. 11

155 Key Points

mTachycardia in adults is defined as a heart rate >100 beats per automaticity (eg, sinus tachycardia), triggered activity outside of . palpitations, light-headedness, syncope, or dyspnea. idely inute. The mechanisms of tachycardia include Increased pacemaker the normal conduction system (eg, ectopic impulses), and reentry Tachycardia can be asymptomatic or associated with The treatment and prognosis of tachycardia vary w depending on the underlying condition. Tachycardia can be associated with a narrow QRS complex Narrow-complex tachycardia can be associated with a regular (<120 ms) or a wide QRS complex (>120 ms). rhythm or an irregular rhythm. mWide-complex tachycardia can associated with uniform QRS Monomorphic wide-complex tachycardia can be associated with orphology (monomorphic) or variable QRS morphology (polymorphic). a regular rhythm or an irregular rhythm. coIdioventricular tachycardia always presents with a wide QRS mplex, whereas SVT can present with a narrow or wide QRS complex.

156

References 1. Mangrum JM, DiMarco JP. The evaluation and management of bradycardia. N Engl J Med. 2000;342(10):703-709.

  1. Berne RML, Levy MN. Physiology. 4th ed. St. Louis, MO: Mosby, Inc.; 1998.
  2. Marino PL. The ICU Book. 3rd ed. Philadelphia, PA: Lippincott Williams & Wilkins—a Wolters Kluwer business; 2007.

  3. Link MS. Clinical practice. Evaluation and initial treatment of supraventricular tachycardia. N Engl J Med. 2012;367(15):1438-1448. Mani BC, Pavri BB. Dual Atrioventricular nodal pathways physiology: a review of

  4. relevant anatomy, electrophysiology, and electrocardiographic manifestations. Indian Pacing Electrophysiol J. 2014;14(1):12-25.

  5. Barold SS, Hayes DL. Non-paroxysmal junctional tachycardia with type I exit block. Heart. 2002;88(3):288.

  6. McCord J, Borzak S. Multifocal atrial tachycardia. Chest. 1998;113(1):203-209.

  7. control in patients with atrial fibrillation. N Engl J Med. 2002;347(23):1825-1833. m 9. electrocardiographic features in 32 patients. N Engl J Med. 1968;279(7):344-349. Wyse DG, Waldo AL, DiMarco JP, et al. A comparison of rate control and rhyth Shine KI, Kastor JA, Yurchak PM. Multifocal atrial tachycardia. Clinical and
  8. Link MS, Berkow LC, Kudenchuk PJ, et al. Part 7: Adult advanced cardiovascular life support: 2015 American Heart Association guidelines update for cardiopulmonary

resuscitation and emergency cardiovascular care. Circulation. 2015;132(18 suppl 2):S444-S464.

  1. Roberts-Thomson KC, Lau DH, Sanders P. The diagnosis.and management of ventricular arrhythmias. Nat Rev Cardiol. 2011;8(6):311-321
  2. Eckardt L, BreithardttG, Kirchhof P. Approach to wide complex tachycardias in patients without struc ural heart disease. Heart. 2006;92(5):704-711. . Brugada P, Brugada J, Mont L, Smeets J, Andries EW. A new approach to the 13differential diagnosis of a regular tachycardia with a wide QRS complex. Circulation. 1991;83(5):1649-1659.

  3. Ip JE, Markowitz SM, Liu CF, Cheung JW, Thomas G, Lerman BB. Differentiating pacemaker-mediated tachycardia from tachycardia due to atrial tracking: utility of V-

A-A-V versus V-A-V response after postventricular atrial refractory period extension. Heart Rhythm. 2011;8(8):1185-1191.

  1. Myerburg RJ, Stewart JW, Hoffman BF. Electrophysiological properties of the canine peripheral A-V conducting system. Circ Res. 1970;26(3):361-378.

  2. Pellegrini CN, Scheinman MM. Clinical management of ventricular tachycardia. Curr Probl Cardiol. 2010;35(9):453-504.